Крок 2 - Медицина 2018 (буклет)

1 / 200
Дитина 5-ти років надійшла до лікарні із скаргами на температуру 38oC та біль у правому боці. Відомо, що хворіє тиждень. Об’єктивно: стан тяжкий, що обумовлено інтоксикацією та дихальною недостатністю за змішаним типом. Перкуторно: праворуч нижче кута лопатки тупий звук, аускульта-тивно: жорстке дихання, під лопаткою праворуч - відсутність дихальних шумів. Лейкоцитоз з нейтрофільним зсуванням ліворуч, підвищена швидкість осідання еритроцитів (ШОЕ). Ваш діагноз: A 5-year-old child came to the hospital with complaints of a temperature of 38oC and pain in the right side. It is known that he has been ill for a week. Objectively: the condition is serious, which caused by intoxication and respiratory insufficiency according to the mixed type. Percussion: a dull sound on the right below the angle of the scapula, auscultation: hard breathing, under the scapula on the right - no breath sounds. Leukocytosis with neutrophil shift to the left, increased erythrocyte sedimentation rate (ESR). Your diagnosis:

Обструктивний бронхіт Obstructive bronchitis

Плеврит Pleuri

Апендицит Appendicitis

Стафілококова деструкція легень Staphylococcal lung destruction

Пневмонія Pneumonia

2 / 200
В природному осередку чуми (Казахстан) виявлено хворого на бубонну форму чуми. Усіх, хто спілкувався з хворим, розмістили в ізоляторі. Що потрібно негайно провести ізольованим? In the natural center of the plague (Kazakhstan), a patient with the bubonic form of the plague was found. Everyone who communicated with the patient was placed in isolation. What should be done immediately in isolation?

Фагопрофілактику Phagoprophylaxis

Введення інтерферону Introduction of interferon

Введення індукторів інтерферону Introduction of interferon inducers

Хіміопрофілактику ремантадином Chemoprophylaxis with rimantadine

Антибіотикопрофілактику Antibiotic prophylaxis

3 / 200
Хвора 65-ти років скаржиться на висип, сильний біль у підлопатковій ділянці справа. Об’єктивно: на шкірі в підлопатковій ділянці справа розміщені лінійно рожево-червоні набряклі вогнища, дещо інфільтровані, з чіткими межами. На поверхні вогнищ згруповані везикули з прозорим вмістом. Поставте діагноз: A 65-year-old patient complains of a rash, severe pain in the subscapular area on the right. Objectively: linear pink-red swollen foci are located on the skin in the subscapular area on the right , somewhat infiltrated, with clear boundaries. Vesicles with transparent contents are grouped on the surface of the foci. Make a diagnosis:

Дерматит Дюрінга During's Dermatitis

Імпетиго Impetigo

Бешиха Бешиха

Оперізувальний лишай (Herpes Zoster) Herpes Zoster

Алергічний дерматит Allergic dermatitis

4 / 200
У хворого, оперованого з приводу гострого парапроктиту, протягом 5-ти діб на тлі антибактеріальної та детоксикуючої терапії, при позитивній динаміці місцевого перебігу захворювання, спостерігаються остуди, гіпертермія, тахікардія, ейфорія. Лікар припускає, що у хворого розвинувся сепсис. Яке дослідження може підтвердити діагноз? A patient operated on for acute paraproctitis, during 5 days on the background of antibacterial and detoxification therapy, with positive dynamics of the local course of the disease, chills, hyperthermia, tachycardia, euphoria. The doctor assumes that the patient has developed sepsis. What research can confirm the diagnosis?

Висів крові на наявність збудника Blood culture for pathogen

Визначення рівня молекул середньої маси Determining the level of average mass molecules

Визначення ступеню мікробної контамінації рани Determining the degree of microbial contamination of the wound

Рентгенографія легень X-ray of lungs

Ультразвукове дослідження печінки Ultrasound examination of the liver

5 / 200
У дівчинки 6-ти років на тлі гострої респіраторної вірусної інфекції (ГРВІ) з’явилися зміни в аналізі сечі: сліди білку, лейкоцити -30-40 в п/з, еритроцити (свіжі) - 10-12 в п/з. Артеріальний тиск - 100/60 мм рт.ст. Який з наведених діагнозів найбільш імовірний? A 6-year-old girl had changes in the urine analysis against the background of an acute respiratory viral infection (ARSI): traces of protein, leukocytes -30-40 in /z, erythrocytes (fresh) - 10-12 in p/z. Blood pressure - 100/60 mm Hg. Which of the following diagnoses is the most probable?

Гострий гломерулонефрит Acute glomerulonephritis

Геморагічний васкуліт Hemorrhagic vasculitis

Інфекція сечовидільної системи Urinary tract infection

Сечокам’яна хвороба Urolithiasis

Вульвовагініт Vulvovaginitis

6 / 200
Серед мешканців присадибних ділянок с. Іванівка, які прилягають до поля агрофірми ’’Дружба” через 1 тиждень після зрошування його пестицидами (гексахлораном) з’явилися симптоми отруєння пестицидами. В результаті лабораторних досліджень вміст гексахлорану в ґрунті сусідніх з полем приватних садиб перевищував ГДК в 2 рази, в повітрі - в 10 разів, у воді криниць (водопостачання населеного пункту є децентралізованим) - в 2 рази. Назвіть вид впливу пестицидів на організм людей, що має місце: Among the residents of the homesteads of the village of Ivanivka, which are adjacent to the field of the agricultural company 'Druzhba', one week after it was irrigated with pesticides (hexachloran), symptoms of pesticide poisoning appeared . As a result of laboratory studies, the content of hexachlorane in the soil of private estates adjacent to the field exceeded the MPC by 2 times, in the air by 10 times, in well water (water supply of the settlement is decentralized) by 2 times. Name the type of impact of pesticides on the human body, which takes place:

Поєднаний Combined

- -

Комбінований Combined

Комплексний Comprehensive

Ізольований Isolated

7 / 200
У дівчини 17-ти років після нервового стресу розвинулася задишка, відчуття ’стороннього тіла” у горлі, колючий та ниючий біль в ділянці верхівки серця. Об’єктивно: плями червоного кольору на обличчі та шиї, дихання везикулярне, частота дихання - 26/хв. Межі серця у нормі, тони чисті, ритмічні, частота серцевих скорочень - 120/хв., артеріальний тиск - 130/80 мм рт.ст. Лікар ”швидкої” вважає, що це соматоформний розлад. Який препарат буде найбільш корисним в даній ситуації? After nervous stress, a 17-year-old girl developed shortness of breath, a feeling of a 'foreign body' in her throat, stabbing and aching pain in the area of the apex of the heart. Objectively: red spots on the face and neck, vesicular breathing, respiratory rate - 26/min. Heart limits are normal, tones are clear, rhythmic, heart rate - 120/min, blood pressure - 130/80 mm Hg. Doctor ' fast' believes that this is a somatoform disorder. Which drug will be most useful in this situation?

Дротаверин Drotaverine

Аспірин Aspirin

Корвалол Corvalol

Парацетамол Paracetamol

Нітрогліцерин Nitroglycerin

8 / 200
В місті функціонує система централізованого господарчо-питного водопостачання. Його джерелом є поверхнева водойма - річка, яка за показниками якості води належить до ІІ-го класу вододжерел. При здійсненні поточного лабораторного контролю за якістю води в точці ’перед надходженням води у зовнішню розподільчу мережу” місцевою СЕС було зафіксовано два поспіль відхилення якості води за показниками епідемічної безпеки. Що найімовірніше є причиною погіршення якості води за епідемічними показниками? The city operates a system of centralized economic and drinking water supply. Its source is a surface reservoir - a river, which according to water quality indicators belongs to the II class of water sources. When implementing of the current laboratory control of water quality at the point 'before the water enters the external distribution network', two consecutive deviations of water quality according to epidemic safety indicators were recorded by the local SES. What is the most likely reason for the deterioration of water quality according to epidemic indicators?

Незадовільне транспортування проби до лабораторії Unsatisfactory sample transportation to the laboratory

Застій води у розподільчій мережі Water stagnation in the distribution network

Порушення правил відбору проб Sampling violation

Погіршення санітарного стану водойми -річки Deterioration of the sanitary state of the reservoir - river

Незадовільна робота очисних (головних) споруд водопроводу Unsatisfactory operation of water treatment (main) facilities

9 / 200
Хвора 37-ми років, хімік-технолог. Турбує поява синців на животі, стегнах, гомілках після невеликих травм або без причини. Під час огляду виявлені численні дрібні та дещо більші синці, які місцями зливаються (від 1х1 до 4х5 см). !нших змін немає. Менструація триває до 7 днів. Селезінка не пальпується. В крові: НЬ- 92 г/л, лейкоцити - 7, 2 • 109/л, тромбоцити - 6 • 109/л, швидкість осідання еритроцитів - 33 мм/год. Який патогенез даного захворювання? A 37-year-old patient, a chemist-technologist. She is concerned about the appearance of bruises on the abdomen, thighs, lower legs after minor injuries or for no reason. During the examination, numerous small and slightly larger bruises that merge in places (from 1x1 to 4x5 cm). There are no other changes. Menstruation lasts up to 7 days. The spleen is not palpable. In the blood: Hb - 92 g/l, leukocytes - 7.2 • 109/l, platelets - 6 • 109/l, erythrocyte sedimentation rate - 33 mm/h. What is the pathogenesis of this disease?

Зниження рівня фібриногену Decrease in fibrinogen

Зниження рівня антигемофільного глобуліну Decrease in antihemophilic globulin

Скорочення тривалості життя тромбоцитів Decreased life expectancy of platelets

Зниження рівня протромбіну крові Decreased blood prothrombin level

Підвищення фібринолітичної активності Increased fibrinolytic activity

10 / 200
Дитині 8 років. Скарги на температуру тіла 39,3oC, головний біль, блювання. 1-й день захворювання. Шкіра чиста. Зів гіперемова-ний. Язик обкладений. Кон’юнктивіт. Склерит. Слабко виражена ригідність потиличних м’язів. Позитивний симптом Керніга справа. Ліквор: цитоз - 340/мкл, лімфоцити - 87%, нейтрофіли - 13%, білок - 140 мг/л. Яким буде найімовірніший діагноз? The child is 8 years old. Complaints about body temperature 39.3oC, headache, vomiting. 1st day of illness. Skin is clean. Pharynx is hyperemic. Tongue coated . Conjunctivitis. Scleritis. Mild rigidity of the occipital muscles. Positive Kernig's sign on the right. CSF: cytosis - 340/μL, lymphocytes - 87%, neutrophils - 13%, protein - 140 mg/L. What would be the most likely diagnosis?

Гнійний менінгіт Suppurative meningitis

Серозний менінгіт Serous meningitis

Пухлина мозку Brain tumor

Арахноїдит Arachnoiditis

Нейротоксикоз Neurotoxicosis

11 / 200
Чотирирічний хлопчик провів цілий день на пляжі. Надвечір у дитини з’явилися головний біль, слабкість, блювання. В ході об’єктивного огляду: обличчя гіперемоване, температура тіла - 38,8oC, частота дихання -28/хв., частота серцевих скорочень - 130/хв. Найбільш імовірною причиною такого стану є: A four-year-old boy spent the whole day at the beach. In the evening, the child developed a headache, weakness, vomiting. During the objective examination: the face is hyperemic, body temperature - 38.8oC, respiratory rate -28/min., heart rate - 130/min. The most likely cause of this condition is:

Сонячний удар Sunstroke

Колапс Collapse

Запаморочення Dizziness

Анафілактичний шок Anaphylactic shock

Синкопальний стан Syncopal state

12 / 200
Дівчинка 13-ти років протягом 5-ти років скаржиться на біль у правому підребер’ї, що віддає у праву лопатку, напади болю пов’язані з порушенням дієти, вони нетривалі, легко знімаються спазмолітичними засобами. Під час нападу болю пальпація живота болюча, максимально в точці проекції жовчного міхура. З найбільшою імовірністю у хворого має місце: A 13-year-old girl has been complaining of pain in the right hypochondrium for 5 years, radiating to the right shoulder blade, pain attacks are associated with a diet violation , they are short-lived, easily removed with antispasmodics. During a pain attack, palpation of the abdomen is painful, maximally at the point of projection of the gallbladder. The patient is most likely to have:

Хронічний панкреатит Chronic pancreatitis

Хронічний холецистит Chronic cholecystitis

Дискінезія жовчовивідних шляхів Biliary tract dyskinesia

Хронічний гастродуоденіт Chronic gastroduodenitis

Виразкова хвороба 12-палої кишки Duodenal ulcer

13 / 200
У студента 18-ти років на шкірі розгинальних поверхонь кінцівок та волосистої частини голови спостерігається розсіяний па-пульозний висип яскраво-червоного кольору, розміром від 1,0 до 5,0 см у діаметрі, поверхня якого у центрі вкрита сріблястими лусочками. Застосовуючи метод зішкрібування, можна виявити симптом ”стеарінової плями” 'термінальної плівки” та ’’кров’яної роси” Який попередній діагноз Вам здається найімовірнішим? An 18-year-old student has a diffuse papular rash of bright red color, size from 1.0 to 5, on the skin of the extensor surfaces of the limbs and scalp .0 cm in diameter, the surface of which is covered with silvery scales in the center. Using the scraping method, the symptom of 'stearin stain' 'terminal film' and 'bloody dew' can be detected. What preliminary diagnosis seems most likely to you?

Папульозно-некротичний туберкульоз шкіри Papular-necrotic tuberculosis of the skin

Червоний плескатий лишай Red lichen

Вторинний сифіліс Secondary syphilis

Атопічний дерматит Atopic dermatitis

Псоріаз Psoriasis

14 / 200
Хвора 65-ти років госпіталізована в реанімаційне відділення в коматозному стані. Хворіє на цукровий діабет II-го типу 10 років. Останні 2 тижні спостерігалися виражена поліурія, полідипсія. Глюкоза сироватки крові -30 ммоль/л, рН артеріальної крові - 7,3. Осмолярність плазми крові - 350 мОсм/л. Хворій встановлено діагноз: діабетична гіперосмолярна кома. Який основний патогенетичний механізм коми? A 65-year-old patient was hospitalized in the intensive care unit in a comatose state. She has been suffering from type II diabetes for 10 years. In the last 2 weeks, severe polyuria and polydipsia were observed. Blood serum glucose - 30 mmol/l, arterial blood pH - 7.3. Blood plasma osmolality - 350 mOsm/l. The patient was diagnosed with diabetic hyperosmolar coma. What is the main pathogenetic mechanism of coma?

Підвищення клубочкової фільтрації Increase glomerular filtration

Гіпокоагуляція Hypocoagulation

Підвищення канальцевої реабсорбції Increasing tubular reabsorption

Гіпонатріємія Hyponatremia

Дегідратація Dehydration

15 / 200
Дільничному лікарю доручили підготувати план проведення комплексу лікувальнопрофілактичних заходів серед населення на підпорядкованій території. Які заходи щодо вторинної профілактики захворювань він повинен внести у цей план? The district doctor was instructed to prepare a plan for carrying out a complex of medical and preventive measures among the population in the subordinate territory. What measures for the secondary prevention of diseases should he include in this plan?

Усунення чинників виникнення хвороб Elimination of factors causing diseases

Попередження виникнення захворювань Disease prevention

Покращання умов життя населення Improving living conditions of the population

Попередження ускладнення хвороб Prevention of complications of diseases

Проведення реабілітаційних заходів Carrying out rehabilitation measures

16 / 200
У 10-річного хлопчика, хворого на гемофілію, спостерігаються явища гострої респіраторної вірусної інфекції з лихоманкою. Який з наведених препаратів, що вводяться із жарознижувальною метою, є протипоказаним цьому хворому? A 10-year-old boy with hemophilia has symptoms of an acute respiratory viral infection with fever. Which of the following antipyretic drugs is contraindicated for this the patient?

Піпольфен Pipolfen

Парацетамол Paracetamol

Ібупрофен Ibuprofen

Ацетилсаліцилова кислота Acetylsalicylic acid

Дротаверин Drotaverin

17 / 200
Дівчинка 14-ти років скаржиться на порушення сну, зниження маси тіла, серцебиття, кардіалгії, стомлюваність. Спостерігається гіперплазія щитоподібної залози II ст., екзофтальм. Які порушення гормонального рівня найбільш характерні для цього захворювання? A 14-year-old girl complains of sleep disturbances, weight loss, palpitations, cardiac pain, fatigue. There is hyperplasia of the thyroid gland of the II stage, exophthalmos. What are the hormonal disorders level most characteristic of this disease?

Підвищення тироксину і трийодтироніну Increase in thyroxine and triiodothyronine

Зниження тироксину Decreased thyroxine

Підвищення рівня йоду, що пов’язаний з білком Elevated protein-bound iodine

Підвищення тиреотропного гормону Increased thyroid hormone

Зниження трийодтироніну Decreased triiodothyronine

18 / 200
Хворий госпіталізований до лікувально-профілактичного закладу з діагнозом: сечокам’яна хвороба. Під час хімічного аналізу було встановлено, що камені складаються з солей сечової кислоти (уратів). Харчування при даній патології має бути скориговано в напрямку зниження в їжі вмісту: The patient was hospitalized in a medical and preventive institution with a diagnosis of urolithiasis. During the chemical analysis, it was established that the stones consist of uric acid salts (urates). Nutrition with this pathology should be adjusted in the direction of reducing the content in food:

Овочів і плодів Vegetables and fruits

М’ясних продуктів Meat products

Зернових продуктів Cereal products

Яєчних продуктів Egg products

Молочних продуктів Dairy products

19 / 200
При клінічному обстеженні дитини спостерігаються різка блідість шкірних покривів із сіруватим відтінком, анемія, поява базофільної зернистості у цитоплазмі поліхроматофільних еритробластів і ретику-лоцитів. У сечі збільшений вміст дельтаамінолевулінової кислоти і копропорфірину. Надмірне надходження якого ксенобіотика в організм дитини призвело до отруєння? During the child's clinical examination, a sharp paleness of the skin with a grayish tint, anemia, the appearance of basophilic granularity in the cytoplasm of polychromatophilic erythroblasts and reticulocytes are observed. The content of delta-aminolevulinic acid is increased in the urine and coproporphyrin. Excessive intake of which xenobiotic in the child's body led to poisoning?

Нітрати Nitrates

Кадмій Cadmium

Миш’як Arsenic

Ртуть Ртуть

Свинець Lead

20 / 200
Хлопчик 10-ти років звернувся до лікаря зі скаргами на загальну слабкість, швидку втомлюваність, дратівливість, зниження працездатності, кровоточивість ясен, петехії на ногах. Недостатність якого вітаміну може мати місце у даному випадку? A 10-year-old boy went to the doctor with complaints of general weakness, quick fatigue, irritability, reduced work capacity, bleeding gums, petechiae on the legs. A deficiency of which vitamin can take place in this case?

Тіамін Thiamine

Аскорбінова кислота Ascorbic acid

Рибофлавін Riboflavin

Вітамін D Vitamin D

Вітамін A Vitamin A

21 / 200
У доношеної дитини на 3-тю добу життя на різних ділянках шкіри виявляються еритема, ерозивні поверхні, тріщини, лущення епідермісу. Немовля виглядає ніби ошпарене окропом. Виявлено позитивний симптом Ні-кольського. Загальний стан дитини тяжкий. Виражений неспокій, гіперестезія, фебриль-на температура. Найімовірніший діагноз у цьому випадку: On the 3rd day of life, a full-term baby shows erythema, erosive surfaces, cracks, flaking of the epidermis on various areas of the skin. The baby looks as if scalded with boiling water. A positive symptom is detected Ni-Kolsky. The general condition of the child is severe. Pronounced restlessness, hyperesthesia, febrile fever. The most likely diagnosis in this case:

Мікотична еритема Mycotic erythema

Ексфоліативний дерматит Exfoliative dermatitis

Флегмона новонародженого Phlegmon of a newborn

Пухирчатка новонародженого Neonatal pemphigus

Псевдофурункульоз Фігнера Fiegner's Pseudofurunculosis

22 / 200
Хлопчик 7-ми років перебуває на лікуванні протягом місяця. При надходженні спостерігалися виражені набряки, протеїнурія - 7,1 г/л, білок в добовій сечі - 4,2 г. В біохімічному аналізі крові утримуються гіпопротеїнемія (43,2 г/л), гіперхолестеринемія (9,2 ммоль/л). Який з перерахованих варіантів гломерулонефриту найімовірніше має місце у хворого? A 7-year-old boy has been receiving treatment for a month. On admission, pronounced edema was observed, proteinuria - 7.1 g/l, protein in daily urine - 4, 2 g. In the biochemical analysis of blood, hypoproteinemia (43.2 g/l), hypercholesterolemia (9.2 mmol/l) are found. Which of the listed variants of glomerulonephritis is most likely to occur in the patient?

Нефротичний Nephrotic

Змішаний Mixed

Іематуричний Iematuric

Нефритичний Jade

Ізольований сечовий Isolated urinary

23 / 200
Хвора 38-ми років звернулася до жіночої консультації зі скаргами на помірні кров’я- нисті виділення зі статевих шляхів, які виникли після затримки чергової менструації на 1,5 місяці. В ході вагінального обстеження: шийка матки не ерозована, симптом ’’зіниці” (+++); матка не збільшена, щільна, рухома, неболюча; придатки з обох боків не збільшені, неболючі; склепіння глибокі. Який діагноз найімовірніший? A 38-year-old patient turned to a women's consultation with complaints of moderate bloody discharge from the genital tract, which occurred after the next menstruation was delayed by 1.5 months. During the vaginal examination: the cervix is not eroded, the 'pupil' symptom (+++); the uterus is not enlarged, dense, mobile, painless; the appendages on both sides are not enlarged, painless; the vaults are deep. What is the most likely diagnosis?

Дисфункціональна маткова кровотеча Dysfunctional uterine bleeding

Позаматкова вагітність Ectopic pregnancy

Внутрішній ендометріоз тіла матки Internal endometriosis of the uterine body

Рак тіла матки Cancer of the uterine body

Маткова вагітність Uterine pregnancy

24 / 200
Тепло, що продукується організмом робітника гарячого цеху в умовах високих температур і зниженої вологості, втрачається переважно випаровуванням, що може призвести до судомної хвороби. Який вид обміну є визначальним при цьому? The heat produced by the body of a hot shop worker in conditions of high temperatures and low humidity is lost mainly by evaporation, which can lead to a convulsive illness. What type of exchange is determining when this?

Білковий Protein

Вуглеводний Carbohydrate

Жировий Fat

Водно-сольовий Water-salt

Вітамінний Vitamin

25 / 200
Хворий 58-ми років скаржиться на здуття кишечника, закрепи, різку слабкість, схуднення. Впродовж останнього місяця випорожнення лише після прийому проносних. Під час пальпації живота спостерігається щільне утворення у лівій здухвинній ділянці. Який найбільш імовірний діагноз? A 58-year-old patient complains of intestinal bloating, constipation, severe weakness, weight loss. During the last month, he has had bowel movements only after taking laxatives. During palpation of the abdomen, a dense mass is observed in the left iliac region. What is the most likely diagnosis?

Рак лівого сечоводу Cancer of the left ureter

Пухлина заочеревинного простору зліва Tumor of the left retroperitoneal space

Пухлина брижі товстої кишки Mesenteric colon tumor

Рак сліпої кишки Cancer of the appendix

Рак сигмоподібної кишки Sigmoid colon cancer

26 / 200
Підліток 16-ти років скаржиться на періодичне виникнення слабкості, запаморочення, відчуття важкості у лівому підребер’ї. Шкірні покриви і видимі слизові оболонки жовтяничні. Баштовий череп. Печінка +2 см, нижній полюс селезінки на рівні пупка. У крові: еритроцити - 2, 7 ■ 1012/л, Hb- 88 г/л, лейкоцити -5, 6 ■ 109 г/л, швидкість осідання еритроцитів -15 мм/год. Вкажіть найбільш імовірну зміну рівня білірубіну у даного хворого: A 16-year-old teenager complains of periodic weakness, dizziness, a feeling of heaviness in the left hypochondrium. The skin and visible mucous membranes are jaundiced. Turret skull. Liver +2 cm, the lower pole of the spleen at the level of the navel. In the blood: erythrocytes - 2.7 ■ 1012/l, Hb - 88 g/l, leukocytes -5.6 ■ 109 g/l, erythrocyte sedimentation rate -15 mm/h Specify the most probable change in the level of bilirubin in this patient:

Підвищення зв’язаного білірубіну Increased bound bilirubin

Підвищення вільного білірубіну Increased free bilirubin

Зниження вільного білірубіну Decreased free bilirubin

Підвищення обох фракцій білірубіну Elevation of both fractions of bilirubin

Зниження зв’язаного білірубіну Decreased bound bilirubin

27 / 200
Жінка 28-ми років звернулася до лікаря зі скаргами на набряклість обличчя, набряки ніг, іноді спостерігає сечу кольору ’м’ясних помиїв”. У підлітковому віці часто хворіла на ангіни. Об’єктивно: бліді шкірні покриви, температура - 36,8oC, пульс - 68/хв., ритмічний. Артеріальний тиск - 170/110 мм рт.ст. Які зміни в сечі найімовірніші? A 28-year-old woman went to the doctor with complaints of swelling of the face, swelling of the legs, sometimes observes urine the color of 'meat slops.' In her teenage years, she was often sick for angina. Objectively: pale skin, temperature - 36.8oC, pulse - 68/min., rhythmic. Blood pressure - 170/110 mm Hg. What changes in urine are most likely?

Підвищення відносної густини, гематурія, бактеріурія Increased relative density, hematuria, bacteriuria

Еритроцитурія й уринозурія Erythrocyturia and urinosuria

Зниження відносної густини, протеїнурія, міоглобінурія Decreased relative density, proteinuria, myoglobinuria

Зниження відносної густини, протеїнурія, бідний сечовий осад Decreased relative density, proteinuria, poor urine sediment

Протеїнурія, гематурія, циліндрурія Proteinuria, hematuria, cylindruria

28 / 200
Хвора 64-х років скаржиться на загальну слабкість, шум у голові, захриплість голосу. Об’єктивно: блідість з жовтяничним відтінком, язик червоний зі згладженими сосочками, асиметрія тактильної і больової чутливості. Пульс - 120/хв., артеріальний тиск - 80/50 мм рт.ст. Пальпується селезінка. У крові: НЬ-58 г/л, еритроцити - 1,2-1012/л, лейкоцити -2,8-109/л, тромбоцити - 140 109/л, швидкість осідання еритроцитів - 17 мм/год, анізоцитоз, пойкілоцитоз - виражені (++). Яке дослідження буде вирішальним у з’ясуванні генезу анемії? A 64-year-old patient complains of general weakness, noise in the head, hoarseness of voice. Objectively: paleness with a jaundiced tint, red tongue with flattened papillae, asymmetry tactile and pain sensitivity. Pulse - 120/min., blood pressure - 80/50 mm Hg. The spleen is palpable. In the blood: Hb-58 g/l, erythrocytes - 1.2-1012/l, leukocytes -2, 8-109/l, platelets - 140 109/l, erythrocyte sedimentation rate - 17 mm/h, anisocytosis, poikilocytosis - pronounced (++). Which study will be decisive in clarifying the genesis of anemia?

Пряма проба Кумбса Direct Coombs test

Фіброгастроскопія Fibrogastroscopy

Люмбальна пункція Lumbar puncture

Непряма проба Кумбса Indirect Coombs Test

Стернальна пункція Sternal puncture

29 / 200
Жінка 43-х років скаржиться на біль у променево-зап’ясткових, гомілковостопних, міжфалангових суглобах кистей та стоп, скутість рухів уранці. Хворіє протягом 7-ми років. Захворювання почалося з симетричного ураження дрібних суглобів кистей та стоп. Спостерігається деформація стоп, їх рухи обмежені та різко болючі. Внутрішні органи без особливостей. Який показник буде найін-формативнішим для постановки діагнозу? A 43-year-old woman complains of pain in the radiocarpal, ankle, interphalangeal joints of the hands and feet, stiffness of movements in the morning. She has been ill for 7 years . The disease began with a symmetrical lesion of the small joints of the hands and feet. Deformation of the feet is observed, their movements are limited and sharply painful. The internal organs are without features. Which indicator will be the most informative for making a diagnosis?

С-реактивний протеїн C-reactive protein

Диспротеїнемія Dysproteinemia

Підвищена швидкість осідання еритроцитів (ШОЕ) Elevated erythrocyte sedimentation rate (ESR)

Ревматоїдний фактор Rheumatoid factor

Лейкоцитоз Leukocytosis

30 / 200
Вам довелося прийти на допомогу жінці 58-ми років, у якої виникла рясна кровотеча з розірваного варикозно розширеного венозного вузла на лівій гомілці. У чому полягає перша допомога? You had to attend to a 58-year-old woman who was bleeding profusely from a ruptured varicose vein in her left leg. What is first aid?

Підвищене положення кінцівки. Стискальна стерильна пов’язка Elevated limb position. Compression sterile bandage

Операція Троянова-Тренделенбурга Troyanov-Trendelenburg operation

Накладання джгута дистально до джерела кровотечі Applying a tourniquet distal to the source of bleeding

Накладання джгута проксимально до джерела кровотечі Applying a tourniquet proximal to the source of bleeding

Z-подібний шов на розірваний варикозний вузол Z-shaped suture on a ruptured varicose node

31 / 200
Чоловік 38-ми років захворів 2 тижні тому, з’явився кашель, слабкість, підвищилася температура до 38,0°C. Стан різко погіршився до кінця 1-го тижня, коли з’явилися остуда, проливний піт, надвечір температура підвищилася до 39,0°C. За 2 дні до госпіталізації у хворого під час кашлю виділилася велика кількість смердючого харкотиння з кров’ю, після чого стан хворого покращився. Пульс -80/хв., частота дихання - 20/хв., температура тіла - 37,6°C. Які зміни можливі на рентгенограмі грудної порожнини? A 38-year-old man fell ill 2 weeks ago, cough, weakness appeared, the temperature rose to 38.0°C. The condition worsened sharply by the end of 1- th week, when chills, profuse sweat appeared, in the evening the temperature rose to 39.0 ° C. 2 days before hospitalization, the patient coughed up a large amount of foul-smelling sputum with blood, after which the patient's condition improved. 80/min., respiratory rate - 20/min, body temperature - 37.6°C. What changes are possible on the X-ray of the chest cavity?

Наявність порожнини з горизонтальним рівнем рідини Presence of a cavity with a horizontal liquid level

Однорідна округла тінь в легеневому полі Uniform round shadow in the lung field

Затемнення частки легені Obscuration of a lobe of the lung

Тінь в нижньому відділі з косою верхньою межею Shadow in the lower section with an oblique upper border

Зміщення середостіння в сторону гомогенної тіні Displacement of the mediastinum towards the homogeneous shadow

32 / 200
Чоловік 39-ти років, який перебував на лікуванні в хірургічному відділенні з приводу гострого холециститу, переведений в терапевтичне відділення з приводу підвищення артеріального тиску до 180/120 мм рт.ст., появи білка в сечі, стійкої лихоманки. Стійкий субфебрилітет був резистентний до антибіо-тикотерапії. Через 10 днів після лікування у хворого виник напад задухи з важким видихом, згодом з’явилися артралгії та еритематозне ураження шкіри. В аналізі крові - 18% еозинофілів. Яке захворювання виникло у хворого? A 39-year-old man, who was treated in the surgical department for acute cholecystitis, was transferred to the therapeutic department due to an increase in blood pressure to 180/120 mm Hg .st., the appearance of protein in the urine, persistent fever. Persistent subfebrility was resistant to antibiotic therapy. 10 days after treatment, the patient had an attack of suffocation with difficult exhalation, later arthralgias and erythematous skin lesions appeared. Blood analysis - 18 % of eosinophils. What disease did the patient have?

Системний червоний вовчак Systemic lupus erythematosus

Глистна інвазія Worm infestation

Неспецифічний аортоартеріїт Nonspecific aortoarteritis

Гострий гломерулонефрит Acute glomerulonephritis

Вузликовий периартеріїт Nodular periarteritis

33 / 200
Хворий 20-ти років доставлений бригадою швидкої медичної допомоги в коматозному стані. Зіниці різко звужені, ціаноз і сухість слизових оболонок, шкіра обличчя і тулуба бліда, волога і холодна. Пульс - 60/хв., артеріальний тиск - 80/60 мм рт.ст., частота дихання - 8-10/хв. Який найбільш імовірний діагноз? A 20-year-old patient was delivered by an ambulance in a comatose state. The pupils are sharply narrowed, cyanosis and dryness of the mucous membranes, the skin of the face and body is pale, moist and cold Pulse - 60/min., blood pressure - 80/60 mm Hg, respiratory rate - 8-10/min. What is the most likely diagnosis?

Отруєння барбітуратами Barbiturate poisoning

Отруєння атропіном Atropine poisoning

Отруєння бензодіазепіновими похідними Poisoning with benzodiazepine derivatives

Отруєння сурогатами алкоголю Alcohol surrogate poisoning

Отруєння опіатами Opiate poisoning

34 / 200
Хлопчик 4-х років госпіталізований до стаціонару зі скаргами на задишку, швидку втомлюваність. В анамнезі часті респіраторні захворювання. Перкуторно: межі серця розширені вліво та вгору. Аускультативно: посилення II тону над легеневою артерією, в IIIII міжребер’ї ліворуч від груднини вислуховується грубий систоло-діастолічний 'машинний” шум, що проводиться в усі інші точки та на спину. Який найбільш імовірний діагноз? A 4-year-old boy was hospitalized with complaints of shortness of breath, rapid fatigue. He has a history of frequent respiratory diseases. Percussion: the borders of the heart are expanded to the left and up. Auscultation: strengthening of the II tone above the pulmonary artery, in the IIIIII intercostal space to the left of the sternum, a coarse systolic-diastolic 'machine' noise is heard, which is carried to all other points and to the back. What is the most likely diagnosis?

Ізольований стеноз отвору легеневої артерії Isolated stenosis of the pulmonary artery opening

Дефект міжпередсердної перегородки Atrial septal defect

Клапанний стеноз аорти Valvular aortic stenosis

Відкрита артеріальна протока Open ductus arteriosus

Дефект міжшлуночкової перегородки Ventricular septal defect

35 / 200
Хворий напередодні відчув легке нездужання, незначний головний біль, слабкість. Сьогодні температура підвищилася до 38,5°C з остудою, значним головним болем, перева- жно в лобовій ділянці, вираженим болем при рухах очних яблук. Шкіра обличчя і кон’юнктива гіперемовані. Виник сухий надсадний кашель. Зів гіперемований, на м’якому піднебінні зерниста енантема, місцями точкові крововиливи. Над легенями дихання з жорстким відтінком. Який найбільш імовірний діагноз? The day before, the patient felt a slight malaise, slight headache, weakness. Today, the temperature rose to 38.5°C with a chill, significant headache, mainly in the frontal in the area, expressed by pain when moving the eyeballs. The skin of the face and conjunctiva are hyperemic. A dry epigastric cough has appeared. The throat is hyperemic, on the soft palate there is granular enanthema, in places point hemorrhages. Over the lungs, breathing with a hard shade. What is the most likely diagnosis?

Черевний тиф Typhoid

Ентеровірусна інфекція Enterovirus infection

Гарячка Ку Ku fever

Висипний тиф Whispering Typhus

Грип Грип

36 / 200
Хвора 36-ти років скаржиться на задишку, відчуття стискання в правій половині грудної клітки, підвищення температури до 38,7oC, кашель з виділенням невеликої кількості слизово-гнійного харкотиння. Хворіє понад тиждень. Скарги пов’язує з переохолодженням. Об’єктивно: легкий акроціаноз губ, пульс ритмічний, 90/хв., артеріальний тиск - 140/85 мм рт.ст. Права половина грудної клітки відстає в акті дихання. Перкусія - справа нижче кута лопатки прослуховує-ться тупість з межею до верху. В цій ділянці дихання відсутнє. Який найбільш імовірний діагноз? A 36-year-old patient complains of shortness of breath, a feeling of tightness in the right half of the chest, an increase in temperature to 38.7oC, a cough with the release of a small amount of mucous-purulent sputum . Has been sick for more than a week. Complaints are related to hypothermia. Objectively: slight acrocyanosis of the lips, rhythmic pulse, 90/min., blood pressure - 140/85 mm Hg. The right half of the chest lags behind in the act of breathing. Percussion - on the right, below the angle of the scapula, dullness is heard with a border to the top. There is no breathing in this area. What is the most likely diagnosis?

Ателектаз легені Atelectasis of the lung

Ексудативний плеврит Exudative pleurisy

Абсцес легені Lung abscess

ТЕЛА BODIES

Позагоспітальна пневмонія Community-acquired pneumonia

37 / 200
До приймального покою швидкою допомогою доставлено хворого 46-ти років зі скаргами на різкий, нападоподібний біль у правій поперековій ділянці, що іррадіює у пахвинну ділянку та на внутрішню поверхню стегна. Біль з’явився раптово кілька годин тому. Напередодні у хворого з’явилася про-фузна безболісна гематурія зі згустками крові черв’якоподібної форми. Раніше нічим не хворів. Яке захворювання можна припустити в першу чергу? A 46-year-old patient was brought to the hospital by ambulance with complaints of sharp, attack-like pain in the right lumbar area, radiating to the inguinal area and to the inner surface of the thigh . The pain appeared suddenly a few hours ago. The day before, the patient had profuse painless hematuria with worm-shaped blood clots. He was not sick before. What disease can be assumed first of all?

Рак правої нирки Right kidney cancer

Некротичний папіліт Necrotic papillitis

Гострий гломерулонефрит Acute glomerulonephritis

Пухлина сечового міхура Bladder tumor

Сечокам’яна хвороба, камінь правої нирки Urolithiasis, right kidney stone

38 / 200
Хвора 37-ми років скаржиться на постійний тупий біль в підребер’ях з іррадіацією в спину, який посилюється після їжі. Турбує здуття живота, часті випорожнення з домішками неперетравленої їжі. Хворіє понад 5 років, схудла на 15 кг. Об’єктивно: помірне здуття живота, болючість в зоні Шоффа-ра, точках Дежардена, Мейо-Робсона. Який з методів дослідження буде найбільш інформативним для підтвердження зовнішньосекре-торної недостатності підшлункової залози? A 37-year-old patient complains of a constant dull pain in the lower ribs with radiation to the back, which intensifies after eating. She is bothered by bloating, frequent bowel movements with impurities of undigested food. She has been sick for more than 5 years, she has lost 15 kg. Objectively: moderate abdominal distension, tenderness in Schoffa's zone, Desjardins, Mayo-Robson points. Which of the research methods will be the most informative to confirm exocrine insufficiency of the pancreas?

Ультразвукове дослідження (УЗД) органів черевної порожнини Ultrasound examination (ultrasound) of abdominal organs

Копрограма Coprogram

Пероральна холецистографія Oral cholecystography

Ретроградна панкреатографія Retrograde Pancreatography

ЕФГДС ЕФГДС

39 / 200
Хворий вводить інсулін в латеральні поверхні плечей, живіт та стегна. Найкомфор-тніше відчуває себе при введенні препарату в живіт. Чим це може бути зумовлено? The patient injects insulin into the lateral surfaces of the shoulders, abdomen, and thighs. He feels most comfortable when the drug is injected into the abdomen. What could be the reason for this?

Меншою кількістю нервових рецепторів Fewer neural receptors

Зручністю введення інсуліну в живіт The convenience of injecting insulin into the stomach

Доступністю введення ін’єкцій Availability of injections

Швидкістю всмоктування інсуліну Insulin absorption rate

Самонавіюванням Spontaneous

40 / 200
Хворий 20-ти років пройшов курс поліхі-міотерапії за схемою ВАМП з приводу гострого лімфобластного лейкозу. Яка морфологічна картина кісткового мозку може свідчити про настання ремісії? A 20-year-old patient underwent a course of polychemotherapy according to the VAMP scheme for acute lymphoblastic leukemia. What morphological picture of the bone marrow can indicate the onset of remission?

Відсутність бластних клітин No blast cells

Вміст бластних клітин до 1% Content of blast cells up to 1%

Вміст бластних клітин до 10% Content of blast cells up to 10%

Вміст бластних клітин до 5% Content of blast cells up to 5%

Вміст бластних клітин до 15 % Content of blast cells up to 15%

41 / 200
Хвора 50-ти років скаржиться на напад болю у правому підребер’ї, блювання з домішками жовчі, впродовж 5-ти років турбував біль у епігастрії, нудота, порушення випорожнення. Об’єктивно: частота серцевих скорочень - 92/хв. Підвищеної повноти, язик обкладений, іктеричні склери. Живіт м’який, болючий у проекції жовчного міхура, локальне м’язове напруження у правому підребер’ї, позитивний симптом Мерфі. В загальному аналізі крові: лейкоцити - 9, 6109/л, швидкість осідання еритроцитів - 14 мм/год. Яке дослідження доцільно призначити для підтвердження діагнозу в даному випадку? A 50-year-old patient complains of an attack of pain in the right hypochondrium, vomiting with bile impurities, for 5 years she has been bothered by pain in the epigastrium, nausea, disturbances defecation. Objectively: heart rate - 92/min. Increased fullness, coated tongue, icteric sclera. Abdomen is soft, painful in the projection of the gallbladder, local muscle tension in the right hypochondrium, positive Murphy's sign. B general blood test: leukocytes - 9, 6109/l, erythrocyte sedimentation rate - 14 mm/h. What research should be prescribed to confirm the diagnosis in this case?

Ультразвукове дослідження жовчного міхура Ultrasound examination of the gallbladder

Бактеріологічне дослідження жовчі Bacteriological examination of bile

Холецистографію Cholecystography

Сцинтіграфію печінки Liver scintigraphy

Ретроградну холангіопанкреатографію Retrograde cholangiopancreatography

42 / 200
Хвора 63-х років оперована з приводу ба-гатовузлового еутиреоїдного зобу великих розмірів. З технічними труднощами вимушено виконана субтотальна резекція обох часток щитоподібної залози. На 4-й день після операції з’явилися судоми м’язів обличчя і верхніх кінцівок, біль в животі. Позитивні симптоми Хвостека і Труссо. Чим найімовірніше зумовлений такий стан хворої? A 63-year-old patient was operated on for a large multinodular euthyroid goiter. With technical difficulties, a subtotal resection of both lobes of the thyroid gland was forced to be performed. On the 4th day after the operation, spasms of the muscles of the face and upper limbs, pain in the abdomen appeared. Positive symptoms of Khvostek and Trousseau. What is the most likely cause of this condition of the patient?

Післяопераційний гіпотиреоз Postoperative hypothyroidism

Трахеомаляція Tracheomalacia

Тиреотоксичний криз Thyrotoxic crisis

Пошкодження зворотнього нерву Injury of the recurrent nerve

Недостатність паращитоподібних залоз Parathyroid insufficiency

43 / 200
Жінка 36-ти років скаржиться на біль, обмеження рухів у дрібних суглобах рук, утруднене ковтання твердої їжі, слабкість, сухий кашель. Об’єктивно: шкіра кистей та передпліч щільна, гладенька. Проксимальні суглоби II-IV пальців кистей рук набряклі, болючі при пальпації. Над легенями сухі розсіяні хрипи, межі серця зміщені вліво на 2 см, тони приглушені. У крові: швидкість осідання еритроцитів - 36 мм/год, y-глобуліни -24%. У сечі: змін немає. Який найбільш імовірний діагноз? A 36-year-old woman complains of pain, limitation of movement in the small joints of the hands, difficulty swallowing solid food, weakness, dry cough. Objectively: the skin of the hands and the forearm is dense, smooth. Proximal joints II-IV of the fingers of the hands are swollen, painful on palpation. Over the lungs, dry scattered rales, the borders of the heart are shifted to the left by 2 cm, tones are muffled. In the blood: erythrocyte sedimentation rate - 36 mm/h, y- globulins -24%. In the urine: there are no changes. What is the most likely diagnosis?

Саркоїдоз Sarcoidosis

Системний червоний вовчак Systemic lupus erythematosus

Дерматоміозит Dermatomyositis

Системна склеродермія Systemic scleroderma

Ревматоїдний артрит Rheumatoid arthritis

44 / 200
Жінка 49-ти років скаржиться на біль у плечових суглобах, що посилюється під час рухів, обмеження рухливості, короткочасну вранішню скутість. Хворіє декілька років. В минулому займалася спортивною гімнастикою. На рентгенограмі плечових суглобів - звуження суглобової щілини, субхондраль-ний остеосклероз, остеофіти у нижньовнутрішньому відділі голівки плечової кістки. Що лежить в основі патогенезу ураження суглобів у хворої? A 49-year-old woman complains of pain in the shoulder joints, which worsens during movements, limitation of mobility, short-term morning stiffness. She has been sick for several years. In the past, she was engaged in sports gymnastics. On the x-ray of the shoulder joints - narrowing of the joint space, subchondral osteosclerosis, osteophytes in the lower inner part of the head of the humerus. What is the basis of the pathogenesis of joint damage in the patient?

Порушення метаболізму хряща Cartilage metabolism disorder

Пошкодження синовіальної оболонки інфекційним агентом Damage of the synovial membrane by an infectious agent

Відкладення у суглобах пірофосфату кальцію Calcium pyrophosphate deposits in the joints

Порушення обміну сечової кислоти Uric acid metabolism disorder

Відкладення у синовіальній оболонці імунних комплексів Deposition of immune complexes in the synovial membrane

45 / 200
Жінка 56-ти років надійшла до клініки зі скаргами на біль та припухлість правого колінного суглоба, які з’явилися тиждень тому. В анамнезі: часті застудні захворювання, хронічний холецистопанкреатит. Об’єктивно: на тильно-бокових поверхнях дистальних між-фалангових суглобів є щільні вузлики невеликих розмірів, злегка болючі при пальпації, спостерігається деформація перших плесно-фалангових суглобів обох стоп, правий колінний суглоб деформований, рухи в ньому обмежені. Який діагноз найбільш імовірний? A 56-year-old woman came to the clinic with complaints of pain and swelling of the right knee joint that appeared a week ago. History: frequent colds, chronic cholecystopancreatitis. Objectively: on the dorso-lateral surfaces of the distal interphalangeal joints, there are dense nodules of small sizes, slightly painful on palpation, deformation of the first metatarsal-phalangeal joints of both feet is observed, the right knee joint is deformed, its movements are limited. What diagnosis is the most likely?

Реактивний артрит Reactive Arthritis

Подагра Gout

Остеоартроз Osteoarthrosis

Ревматичний поліартрит Rheumatoid polyarthritis

Ревматоїдний артрит Rheumatoid arthritis

46 / 200
Чоловік 47-ми років страждає на рак легень в термінальній стадії. Розповідає, що протягом останніх 2-3-х тижнів помітив зміни в своєму психічному стані: бачить ввечері покійну матір, яка кличе до себе, відчуває запах сирої землі, гнилого листя, бачить незнайомих людей, які несуть труну. При цьому відчуває страх. Які симптоми психічних розладів у хворого? A 47-year-old man suffers from lung cancer in the terminal stage. He says that during the last 2-3 weeks he has noticed changes in his mental state: he sees in the evening the deceased mother, who calls to her, smells raw earth, rotten leaves, sees strangers carrying a coffin. At the same time, she feels fear. What are the symptoms of mental disorders in the patient?

Депресивні розлади Depressive disorders

Ілюзорні розлади Illusory disorders

Психосенсорні розлади Psychosensory disorders

Маячні ідеї Delusive Ideas

Справжні галюцинації True hallucinations

47 / 200
Чоловік 38-ми років надійшов до лікарні у непритомному стані. Захворів учора: з’явився головний біль, нудота, блювання, температура тіла - 38,5°С, виникло запаморочення, марення. Протягом останніх 4-х днів скаржився на біль та зниження слуху на ліве вухо. Об’єктивно: стан сопорозний, ригідність потиличних м’язів, симптом Керніга з обох боків, загальна гіперстезія, гноєтеча з лівого вуха. Який з перерахованих діагнозів найбільш імовірний? A 38-year-old man came to the hospital in an unconscious state. He fell ill yesterday: headache, nausea, vomiting, body temperature - 38.5°C , dizziness, delirium appeared. During the last 4 days, he complained of pain and hearing loss in the left ear. Objectively: a soporose state, stiffness of the occipital muscles, Kernig's symptom on both sides, general hyperesthesia, purulent discharge from the left ear. What which of the listed diagnoses is the most likely?

Паренхіматозно-субарахноїдальний крововилив Parenchymatous-subarachnoid hemorrhage

Субарахноїдальний крововилив Subarachnoid hemorrhage

Вторинний гнійний менінгіт Secondary purulent meningitis

Туберкульозний менінгіт Tuberculous meningitis

Первинній гнійний менінгіт Primary purulent meningitis

48 / 200
Внаслідок дорожньо-транспортної пригоди потерпілий отримав множинні переломи кінцівок та кісток тазу. В анамнезі: гемофілія А. Під час обстеження формуються гематоми на ушкоджених ділянках. Стан погіршується. Артеріальний тиск - 90/50 мм рт.ст. Вкажіть найбільш доцільну комбінацію ін-фузійних засобів для лікування пацієнта після застосування поліглюкіну та сольових розчинів: As a result of a traffic accident, the victim received multiple fractures of the limbs and pelvic bones. History: hemophilia A. During the examination, hematomas form on the damaged areas. The condition worsens. Arterial pressure - 90/50 mm Hg Specify the most appropriate combination of infusion agents for the treatment of the patient after the use of polyglukin and saline solutions:

Кріопреципітат, еритроцитарна маса Cryoprecipitate, erythrocyte mass

Свіжозаморожена плазма, альбумін Fresh-frozen plasma, albumin

Еритроцитарна маса, альбумін Erythrocyte mass, albumin

Еритроцитарна маса Erythrocyte mass

Кріопреципітат, глюкоза Cryoprecipitate, glucose

49 / 200
Чоловік 74-х років скаржиться на періодичне запаморочення, порушення мови, втрату пам’яті на поточні події, повільну ходу. Хворіє близько 5 років. Об’єктивно: мова дизартрична, міміка бідна, сутулувата постава. Сухожилкові рефлекси S>D, симптом Бабінського двобічно, симптоми орального автоматизму, м’язовий тонус підвищений за пластичним типом. В позі Ромберга нестійкий, ністагм горизонтальний. Який діагноз найбільш імовірний? A 74-year-old man complains of periodic dizziness, slurred speech, loss of memory for current events, slow gait. He has been ill for about 5 years. Objectively: speech is dysarthric, facial expression poor, stooped posture. Tendon reflexes S>D, Babinsky's symptom bilaterally, symptoms of oral automatism, muscle tone is increased according to the plastic type. Romberg's position is unstable, nystagmus is horizontal. What is the most likely diagnosis?

Набряк мозку Brain edema

Менінгоенцефаліт Meningoencephalitis

Дисциркуляторна енцефалопатія Dyscirculatory encephalopathy

Енцефаломієліт Encephalomyelitis

Хвороба Паркінсона Parkinson's disease

50 / 200
Чоловік 70-ти років хворіє на ішемічну хворобу серця. Настрій помітно знижений, тривожний. На тлі тривалого безсоння з’явилися страхи, небажання жити, думки покінчити з собою. Подовгу сидить у одній і тій самій позі, відповідає не одразу, тихо, монотонним голосом. На обличчі вираз страждання, болю, страху. Який провідний психопатологічний синдром? A 70-year-old man suffers from coronary heart disease. His mood is markedly depressed, anxious. Against the background of prolonged insomnia, fears, unwillingness to live, thoughts of suicide appeared . Sits for a long time in the same position, does not answer immediately, quietly, in a monotonous voice. The expression of suffering, pain, fear on the face. What is the leading psychopathological syndrome?

Астенічний синдром Asthenic syndrome

Обсесивний синдром Obsession Syndrome

Фобічний синдром Phobic syndrome

Параноїдний синдром Paranoid Syndrome

Депресивний синдром Depressive syndrome

51 / 200
Жінка 28-ми років протягом 12-ти років хворіє на хронічний гломерулонефрит, який весь цей час мав латентний перебіг. За остан- ні півроку виникла загальна слабкість, зниження апетиту, працездатності, нудота. Хвора скаржиться на головний 6іль, 6іль у суглобах. При обстеженні: анемія, підвищення рівня сечовини крові до 34,5 ммоль/л, креати-нін крові - 0,766 ммоль/л, гіперкаліємія. Чим ускладнився перебіг хвороби? A 28-year-old woman has been suffering from chronic glomerulonephritis for 12 years, which had a latent course all this time. Over the last six months, she developed general weakness, decreased appetite, work capacity, nausea. The patient complains of headache, pain in the joints. During the examination: anemia, blood urea level increased to 34.5 mmol/l, blood creatinine - 0.766 mmol/l, hyperkalemia. What complicated the course of the disease?

Амілоїдозом нирок Kidney amyloidosis

Пієлонефритом Pyelonephritis

Гострою нирковою недостатністю Acute kidney failure

Нефротичним синдромом Nephrotic Syndrome

Хронічною нирковою недостатністю Chronic renal failure

52 / 200
Хворий 70-ти років скаржиться на слабкість, запаморочення, короткочасні періоди непритомності, біль в ділянці серця. Під час аускультації серця: частота серцевих скорочень - 40/хв., тони ритмічні, I тон періодично значно посилений, артеріальний тиск -160/90 мм рт.ст. Яка найбільш імовірна причина ге-модинамічних порушень? A 70-year-old patient complains of weakness, dizziness, short-term periods of unconsciousness, pain in the heart area. During auscultation of the heart: heart rate - 40/min. , tones are rhythmic, I tone is periodically significantly increased, blood pressure -160/90 mm Hg. What is the most likely cause of hemodynamic disturbances?

Повна блокада лівої ніжки пучка Гіса Complete blockade of the left leg of the bundle of His

АВ блокада I ступеня 1st stage AV block

АВ блокада III ступеня AV block III degree

Синусова брадикардія Sinus bradycardia

Брадисистолічна форма миготливої аритмії Bradysystolic form of atrial fibrillation

53 / 200
У хворого 35-ти років, що страждає на хронічний гломерулонефрит і останні 3 роки перебуває на гемодіалізі, з’явилися перебої в роботі серця, гіпотонія, зростаюча слабкість, задишка. На ЕКГ: брадикардія, атріовентри-кулярна блокада I ст., високі загострені зубці Т Напередодні - грубе порушення питного та дієтичного режимів. Яка найбільш імовірна причина вказаних змін? A 35-year-old patient who suffers from chronic glomerulonephritis and has been on hemodialysis for the past 3 years has had heart failure, hypotension, increasing weakness, shortness of breath. On the ECG: bradycardia, atrioventricular blockade of the 1st degree, high pointed T waves The day before - a gross violation of the drinking and dietary regimes. What is the most likely cause of the indicated changes?

Гіпернатріємія Hypernatremia

Гіпокаліємія Hypokalemia

Гіпокальціємія Hypocalcemia

Гіперкаліємія Hyperkalemia

Гіпергідратація Hyperhydration

54 / 200
Хворий на виразкову хворобу 12-палої кишки звернувся зі скаргами на слабкість, задишку при незначному навантаженні, бажання їсти крейду. Об’єктивно: блідий, трофічні зміни шкіри. В крові: Hb- 82 г/л; еритроцити - 3, 3 • 1012/л; КП- 0,75, ретикулоцити - 0,7%, залізо крові - 5,6 мкмоль/л. Яке ускладнення розвинулося у хворого? A patient with peptic ulcer disease of the duodenum complained of weakness, shortness of breath with slight exertion, a desire to eat chalk. Objectively: pale, trophic skin changes. In the blood: Hb - 82 g/l; erythrocytes - 3.3 • 1012/l; CP - 0.75, reticulocytes - 0.7%, blood iron - 5.6 μmol/l. What complication developed in the patient?

Апластична анемія Aplastic anemia

Гемолітична анемія Hemolytic anemia

Еритремія Erythremia

Б12-фолієводефіцитна анемія B12-folate deficiency anemia

Залізодефіцитна анемія Iron deficiency anemia

55 / 200
Дитина 1,5 років народилася з масою тіла 3100 г, довжиною 5І см. Годувалася грудьми. Після введення прикорму (вівсяна каша) перестала набирати вагу, з’явилися випорожнення з неприємним запахом у великій кількості. Об’єктивно: явища гіпотрофії II ст., блідість шкіри, великий живіт. Який найбільш імовірний діагноз? A 1.5-year-old child was born with a body weight of 3100 g, a length of 51 cm. She was breast-fed. After the introduction of complementary food (oatmeal), she stopped gaining weight, appeared stools with an unpleasant smell in large quantities. Objectively: phenomena of hypotrophy of the II century, pale skin, large abdomen. What is the most likely diagnosis?

Целіакія Celiac

Муковісцидоз Cystic Fibrosis

Проста диспепсія Simple dyspepsia

Кишкова інфекція Intestinal infection

Глистна інвазія Worm infestation

56 / 200
Хвора 36-ти років захворіла гостро. Захворювання почалося з високої температури тіла, остуди, профузного поту. З’явився тупий біль у поперековому відділі, неприємні відчуття при сечовипусканні. Об’єктивно: напруження м’язів поперекового відділу, позитивний симптом постукування з обох боків. В крові: лейкоцити до 12 • 109/л, нейтрофільоз. В сечі: білок - 0,6 г/л, лейкоцити на все п/з, бактерії - більше 100 000 в 1 мл сечі. Яким буде попередній діагноз? A 36-year-old patient became acutely ill. The illness began with high body temperature, chills, profuse sweat. Dull pain appeared in the lumbar region, unpleasant sensations during urination . Objectively: tension in the muscles of the lumbar region, a positive symptom of tapping on both sides. In the blood: leukocytes up to 12 • 109/l, neutrophilia. In the urine: protein - 0.6 g/l, leukocytes for all p/z , bacteria - more than 100,000 in 1 ml of urine. What will be the preliminary diagnosis?

Гострий пієлонефрит Acute pyelonephritis

Сечокам’яна хвороба Urolithiasis

Гострий гломерулонефрит Acute glomerulonephritis

Туберкульоз нирки Kidney tuberculosis

Гострий цистит Hostry cystitis

57 / 200
Хворий 46-ти років вперше помітив набряки на ногах, слабкість, відчуття 'повноти” та важкості в правому підребер’ї. 20 років хворіє на ревматоїдний артрит. Збільшені печінка і селезінка, щільної консистенції. Креатинін крові - 0,23 ммоль/л, протеїнемія - 53 г/л, холестерин - 4,2 ммоль/л, питома вага сечі -1012, протеїнурія - 3,3 г/л, поодинокі воскоподібні циліндри, еритроцити вилужені в п/з, лейкоцити - 5-6 в п/з. Яке ускладнення найбільш імовірне? A 46-year-old patient first noticed swelling in the legs, weakness, a feeling of 'fullness' and heaviness in the right hypochondrium. He has been suffering from rheumatoid arthritis for 20 years. Enlarged liver and spleen, dense consistency. Blood creatinine - 0.23 mmol/l, proteinemia - 53 g/l, cholesterol - 4.2 mmol/l, urine specific gravity -1012, proteinuria - 3.3 g/l, single waxy cylinders, erythrocytes leached out in p/z, leukocytes - 5-6 in p/z. What complication is most likely?

Серцева недостатність Heart failure

Амілоїдоз нирок Kidney amyloidosis

Хронічний пієлонефрит Chronic pyelonephritis

Гострий гломерулонефрит Acute glomerulonephritis

Хронічний гломерулонефрит Chronic glomerulonephritis

58 / 200
У дівчинки 12-ти років хронічний вірусний гепатит С впродовж 7-ми років. Стан погіршився за останні 6 місяців. Схудла, з’явилася жовтяничність шкірних покривів і склер. Виражене свербіння шкіри, судинні 'зірочки” носові кровотечі. Асцит. Печінка +4 см, селезінка на рівні пупкової лінії. З розвитком якого синдрому пов’язане виражене свербіння шкіри у хворої? A 12-year-old girl has chronic viral hepatitis C for 7 years. The condition has worsened over the past 6 months. She has lost weight, jaundice of the skin and sclera has appeared Pronounced itching of the skin, vascular 'stars' nosebleeds. Ascites. Liver +4 cm, spleen at the level of the umbilical line. The development of which syndrome is associated with the development of pronounced itching of the skin in the patient?

Холестаз Cholestasis

Портальна гіпертензія Portal hypertension

Гіперспленізм Hypersplenism

Набряково-асцитичний синдром Ascitic Ascitic Syndrome

Гепатоцелюлярна недостатність Hepatocellular failure

59 / 200
Спортсмен 26-ти років впав з упором на правий плечовий суглоб. Зазначає різкий біль, обмеження пасивних рухів, активні рухи в плечовому суглобі неможливі. Відведену і зігнуту у плечовому суглобі травмовану руку підтримує здоровою. Об’єктивно: в ділянці дельтоподібного м’яза западина, під шкірою різко випинається акроміальний відросток лопатки, голівка плечової кістки пальпується під дзьобоподібним відростком. Який найбільш імовірний клінічний діагноз? A 26-year-old athlete fell with an emphasis on the right shoulder joint. He notes sharp pain, limitation of passive movements, active movements in the shoulder joint are impossible. Abducted and bent shoulder the joint keeps the injured hand healthy. Objectively: there is a depression in the area of the deltoid muscle, the acromial process of the scapula protrudes sharply under the skin, the head of the humerus can be palpated under the beak-like process. What is the most likely clinical diagnosis?

Вивих плеча Shoulder dislocation

Перелом анатомічної шийки плеча Fracture of the anatomical neck of the humerus

Відрив великого пагорбка плечової кістки Detachment of the greater tubercle of the humerus

Перелом голівки плечової кістки Fracture of humeral head

Перелом хірургічної шийки плеча Fracture of the surgical neck of the shoulder

60 / 200
Хворий 30-ти років працює клепальником протягом 6-ти років. Скарги на різкий біль у плечовому поясі, особливо в нічний час. Пальці рук німіють та біліють при охолодженні. Став дратівливим. Об’єктивно: кисті набряклі, холодні, ціанотичні. Різко знижені всі види чутливості (больова, температурна, вібраційна). При пробі на холод - симптом 'мертвих пальців’.’ Відзначається слабкість у привідних м’язах V пальця; змінена електрозбудженість. Сухожилкові та періостальні рефлекси живі. Який попередній діагноз? A 30-year-old patient has been working as a riveter for 6 years. Complains of sharp pain in the shoulder girdle, especially at night. The fingers are numb and turn white when cooled . Became irritable. Objectively: the hands are swollen, cold, cyanotic. All types of sensitivity (pain, temperature, vibration) are sharply reduced. When tested for cold - the symptom of 'dead fingers.' There is weakness in the adductor muscles of the V finger; electrical excitability has changed. Tendon and periosteal reflexes are alive. What is the previous diagnosis?

Вібраційна хвороба загальної дії Vibration disease of general effect

Вузликовий периартеріїт Nodular periarteritis

Вібраційна хвороба локальної дії Vibration disease of local effect

Вегетативно-сенсорна полінейропатія Vegetative-sensory polyneuropathy

Синдром Рейно Raynaud's

61 / 200
Чоловік 31-го року багато років лікується у психіатра. На тлі тривалого безсоння з’явилися страхи, думки покінчити з собою, намагався повіситися. Настрій помітно знижений, від лікування відмовляється. Які дії є найбільш рекомендованими для профілактики суїциду хворого? A 31-year-old man has been treated by a psychiatrist for many years. Against the background of prolonged insomnia, fears appeared, thoughts of suicide, he tried to hang himself. His mood is markedly depressed, from treatment is refused. What actions are the most recommended for the prevention of a patient's suicide?

Госпіталізація в неврологічне відділення Hospitalization in the neurological department

Госпіталізація в психлікарню Hospitalization in a mental hospital

Суворий нагляд вдома Strict supervision at home

Амбулаторне лікування Outpatient treatment

Психотерапевтична бесіда Psychotherapeutic conversation

62 / 200
Дитина 7-ми місяців доставлена у хірургічне відділення через 8 годин після захворювання зі скаргами на напади неспокою, біль у животі, одноразове блювання. Об’єктивно: у правій половині живота пальпується пухлиноподібне утворення. При ректальному дослідженні - кров у вигляді ’малинового желе” Яке захворювання можна припустити? A 7-month-old child was brought to the surgical department 8 hours after the illness with complaints of anxiety attacks, abdominal pain, one-time vomiting. Objectively: in the right a tumor-like formation is palpated in half of the abdomen. During rectal examination - blood in the form of 'raspberry jelly' What disease can be assumed?

Ентерокістома Enterocystoma

Пухлина черевної порожнини Abdominal tumor

Подвоєння кишечника Intestinal doubling

Глистна інвазія Worm infestation

Інвагінація Intussusception

63 / 200
У хворої 25-ти років відсутні менструації впродовж 6-ти місяців. Менструації з 16 років по 3-5 днів із затримками на місяць, помірні, болючі. У жінки надмірна вага тіла, гірсутизм, неплідність. При бімануальному дослідженні тіло матки нормальних розмірів. З обох сторін від матки пальпуються збільшені в розмірах яєчники до 4 см в діаметрі, щільної консистенції, безболісні, рухомі при зсуві. Яким буде діагноз? A 25-year-old patient has had no menstruation for 6 months. Menstruation since the age of 16 is 3-5 days with delays per month, moderate, painful. women overweight, hirsutism, infertility. On bimanual examination, the body of the uterus is of normal size. On both sides of the uterus, enlarged ovaries up to 4 cm in diameter, dense consistency, painless, mobile when shifted are palpable. What will be the diagnosis?

Хронічний двосторонній аднексит Chronic bilateral adnexitis

Синдром Шихана Sheehan Syndrome

Дермоїдні кісти яєчників Ovarian dermoid cysts

Синдром галактореї та аменореї Syndrome of galactorrhea and amenorrhea

Полікістоз яєчників Polycystic ovary

64 / 200
У дівчинки 3-х місяців діарея та гіпотро- фії. Народилася з масою 3300 г. З 2-х місяців переведена на штучне вигодовування коров’ячим молоком. Шкіра суха, м’язовий тонус знижений, випорожнення рідкі, зеленуваті, смердючі. У копрограмі багато нейтрального жиру. Хлориди поту - 70 мекв/л. Яке захворювання найбільш імовірне? A 3-month-old girl has diarrhea and hypotrophy. She was born with a weight of 3300 g. From 2 months, she was transferred to artificial feeding with cow's milk. Skin dry, muscle tone is reduced, stools are liquid, greenish, smelly. There is a lot of neutral fat in the coprogram. Sweat chloride - 70 meq/l. What disease is most likely?

Муковісцидоз Cystic Fibrosis

Кишкова інфекція Intestinal infection

Непереносимість білку коров’ячого молока Intolerance to cow's milk protein

Целіакія Celiac

Дисахаридна недостатність Disaccharide deficiency

65 / 200
Для оцінки популяційного здоров’я дитячого населення були використані наступні показники: поширеність хвороб, первинна захворюваність, структура захворюваності, патологічна ураженість та інвалідність. Який з цих показників відображає рівень захворюваності дітей лише на хронічні хвороби та морфофункціональні порушення? The following indicators were used to assess the population health of the child population: prevalence of diseases, primary morbidity, structure of morbidity, pathological damage and disability. Which of these indicators reflects the level morbidity of children only for chronic diseases and morphofunctional disorders?

Загальна захворюваність Total morbidity

Патологічна ураженість Pathological lesion

Первинна захворюваність Primary morbidity

Поширеність Prevalence

Інвалідність Disability

66 / 200
Робітник працює у ливарні з кольоровими металами та сплавами протягом 12-ти років. У повітрі робочої зони зареєстровано підвищений вміст важких металів, оксидів вуглецю та азоту. Під час регулярного медичного огляду у хворого виявлено астено-вегетативний синдром, різкий біль в животі, закрепи, біль в ділянці печінки. У сечі: амі-нолевулінова кислота та копропорфірін. В крові: ретикулоцитоз, зниження рівню гемоглобіну. Причиною такої інтоксикації є: A worker has been working in a foundry with non-ferrous metals and alloys for 12 years. In the air of the working area, an increased content of heavy metals, carbon and nitrogen oxides was registered. During regular medical examination of the patient revealed an astheno-vegetative syndrome, sharp pain in the abdomen, constipation, pain in the liver area. In the urine: aminolevulinic acid and coproporphyrin. In the blood: reticulocytosis, a decrease in the level of hemoglobin. The cause of such intoxication is:

Олово Lead

Цинк Цинк

Оксид вуглецю Carbon monoxide

Оксид азоту Nitrogen Oxide

Свинець та його солі Lead and its salts

67 / 200
У дитини 13-ти років сполучена вада мі-трального клапана з переважанням недостатності. На тлі гострої респіраторної вірусної інфекції (ГРВІ) підсилилися задишка та загальна слабкість, з’явилося відчуття стискання в грудях, сухий кашель. В легенях вислуховуються різнокаліберні вологі хрипи. Печінка біля краю реберної дуги. Який невідкладний стан розвинувся у дитини? A 13-year-old child has a combined defect of the mitral valve with a predominance of insufficiency. Against the background of an acute respiratory viral infection (ARSI), shortness of breath and general weakness increased, with' there was a feeling of tightness in the chest, a dry cough. Wet rales of various caliber are heard in the lungs. The liver is near the edge of the costal arch. What emergency condition has developed in the child?

Гостра лівошлуночкова серцева недостатність Acute left ventricular heart failure

Гостра тотальна серцева недостатність Acute total heart failure

Гостра правошлуночкова серцева недостатність Acute right ventricular heart failure

Гостра судинна недостатність Acute vascular insufficiency

Хронічна лівошлуночкова серцева недостатність Chronic left ventricular heart failure

68 / 200
В терапевтичному відділенні стаціонару перебувало на лікуванні 50 хворих, 40 із них працюють. Перед випискою зі стаціонару працюючим видавалися листки непрацездатності. Хто має право підписувати листки непрацездатності? In the therapeutic department of the hospital, 50 patients were treated, 40 of them are working. Before discharge from the hospital, sick leave was issued to the workers. Who has the right to sign sick leave?

Лікар-куратор, завідувач відділєння Supervising physician, head of department

Головний лікар, лікар-куратор Chief physician, supervising physician

- -

Головний лікар, завідувач відділення Chief physician, department head

Завідувач відділення, заступник головного лікаря з медичної роботи Head of department, deputy chief physician for medical work

69 / 200
Жінка 22-х років звернулася до жіночої консультації з приводу вагітності 11-12 тижнів. Під час обстеження виявлено позитивну реакцію Васермана. Дерматолог діагностував вторинний латентний сифіліс. Яка тактика ведення даної вагітності? A 22-year-old woman turned to a women's consultation about a pregnancy of 11-12 weeks. During the examination, a positive Wasserman reaction was found. The dermatologist diagnosed secondary latent syphilis. What tactics management of this pregnancy?

Протисифілітичне лікування тричі протягом вагітності Antisyphilitic treatment three times during pregnancy

Штучне переривання вагітності після курсу протисифілітичної терапії Artificial termination of pregnancy after a course of antisyphilitic therapy

Штучне переривання вагітності до курсу протисифілітичної терапії Artificial termination of pregnancy before the course of antisyphilitic therapy

Пролонгування вагітності після першого курсу протисифілітичної терапії Prolongation of pregnancy after the first course of antisyphilitic therapy

Термінове переривання вагітності Emergency termination of pregnancy

70 / 200
Серед причин смертності населення України переважають нещасні випадки, отруєння та травми, злоякісні новоутворення, хвороби системи кровообігу, ендокринні, органів дихання тощо. Назвіть клас хвороб, що посідає перше рангове місце: Among the causes of mortality of the population of Ukraine, accidents, poisoning and injuries, malignant neoplasms, diseases of the circulatory system, endocrine, respiratory organs, etc. prevail. Name the class of diseases that ranks first ranking place:

Хвороби органів дихання Diseases of respiratory organs

Хвороби системи кровообігу Diseases of the circulatory system

Ендокринні захворювання Endocrine diseases

Нещасні випадки, отруєння та травми Accidents, poisonings and injuries

Злоякісні новоутворення Malignant neoplasms

71 / 200
Дитина 3-х років хворіє 3 доби. Спостерігаються температура, кашель, нежить, кон’юнктивіт, світлобоязнь, білуватий крапковий висип на слизовій оболонці ясен та внутрішній поверхні щік і плямиста енантема твердого та м’якого піднебіння. Яке захворювання можна діагностувати? A 3-year-old child has been sick for 3 days. Fever, cough, runny nose, conjunctivitis, photophobia, whitish dotted rash on the mucous membrane of the gums and the inner surface of the cheeks are observed and mottled enanthema of the hard and soft palate. What disease can be diagnosed?

Кір Кір

Герпетична інфекція Herpes infection

Ентеровірусна інфекція Enterovirus infection

Інфекційний мононуклеоз Infectious mononucleosis

Гостра респіраторна вірусна інфекція Acute respiratory viral infection

72 / 200
У дитини 10-ти років скарги на біль і припухлість колінних і гомілковостопних суглобів та біль у шийному відділі хребта. Ранкова скутість. Призначена протизапальна терапія дала позитивний терапевтичний ефект. Яке захворювання можна припустити? A 10-year-old child complains of pain and swelling of the knee and ankle joints and pain in the cervical spine. Morning stiffness. The prescribed anti-inflammatory therapy had a positive therapeutic effect. What disease can be assumed?

Остеохондроз Osteochondrosis

Ювенільний ревматоїдний артрит Juvenile rheumatoid arthritis

Ревматизм Rheumatism

Синдром Рейтера Reiter's Syndrome

Інфекційний артрит Infectious arthritis

73 / 200
Дитині 5 років. 2 тижні тому перенесла ангіну. Скарги матері на поганий апетит, порушений сон. При об’єктивному дослідженні виявлена частота серцевих скорочень -100/хв. Як слід оцінити цей показник? The child is 5 years old. She had angina 2 weeks ago. The mother complains of poor appetite, disturbed sleep. An objective examination revealed a heart rate of -100/min. How should this indicator be evaluated?

Дихальна аритмія Respiratory arrhythmia

Вікова норма Age norm

Брадикардія Bradycardia

Тахікардія Tachycardia

Пароксизмальна тахікардія Paroxysmal tachycardia

74 / 200
У жіночу консультацію звернулася хвора 36-ти років зі скаргами на підвищену дратівливість, плаксивість, головний біль, серцебиття, набряки на руках і ногах, зменшення сечовиділення, нагрубання молочних залоз. Ці прояви виникають і поступово зростають за кілька днів до менструації і зникають з її початком. Менструальний цикл без порушень. Перераховані скарги з’явилися впродовж останнього року. Поставте діагноз: A 36-year-old patient came to the women's clinic with complaints of increased irritability, tearfulness, headache, palpitations, swelling of the hands and feet, decreased urination, thickening of milk glands. These manifestations appear and gradually increase a few days before menstruation and disappear with its onset. Menstrual cycle without disturbances. The listed complaints appeared during the last year. Make a diagnosis:

Передменструальний синдром Premenstrual syndrome

Синдром Шихана Sheehan Syndrome

Клімактеричний синдром Climacteric syndrome

Синдром Штейна-Левенталя Stein-Leventhal Syndrome

Адреногенітальний синдром Adrenogenital syndrome

75 / 200
Хворий 67-ми років скаржиться на задишку, біль у грудях, загальну слабкість. Хворіє 5 місяців. Об’єктивно: температура тіла - 37,3°Є, пульс - 96/хв. Над правою легенею голосове тремтіння не визначається, перку-торний звук тупий, дихання не прослуховується. У харкотинні - домішка крові, дифузно змішаної зі слизом. Який найбільш імовірний діагноз? A 67-year-old patient complains of shortness of breath, chest pain, general weakness. He has been ill for 5 months. Objectively: body temperature - 37.3°С, pulse - 96/min. Over the right lung, the voice tremor is not determined, the percussive sound is dull, breathing is not heard. In the sputum there is an admixture of blood diffusely mixed with mucus. What is the most likely diagnosis?

Великовогнищева пневмонія Large focus pneumonia

Ексудативний плеврит Exudative pleurisy

Бронхоектатична хвороба Bronchoectatic disease

Вогнищевий туберкульоз легень Focal pulmonary tuberculosis

Рак легень Lung cancer

76 / 200
Недостатньо очищені відходи промислового підприємства скидаються у річку, вода з якої використовується для господарсько-питного водопостачання. Це спричиняє загибель деяких мікроорганізмів, порушення процесів самоочищення води та погіршення її якості, що може мати негативний вплив на стан здоров’я людей. Така дія факторів навколишнього середовища називається: Insufficiently purified waste from an industrial enterprise is discharged into a river, the water from which is used for domestic and drinking water supply. This causes the death of some microorganisms, disruption of water self-purification processes and deterioration of its quality , which can have a negative impact on people's health. This effect of environmental factors is called:

Комплексна Complex

Пряма Direct

Непряма Indirect

Поєднана Combined

Комбінована Combined

77 / 200
Хворий 48-ми років госпіталізований з нападами судом. Багато років страждає на виразку 12-ти палої кишки (ДПК). Протягом останнього місяця спостерігає щоденне блювання, схуд на 20 кг. Об’єктивно: хворий виснажений. В епігастральній ділянці визначається ”шум плеску’.’ Нижня межа шлунка на рівні гребінцевої лінії. Лабораторно: загальний білок - 47 г/л; K - 2,1 ммоль/л, Na -118 ммоль/л, Ca - 1,6 ммоль/л, хлориди - 82 ммоль/л, гематокрит - 64%. Який попередній діагноз? A 48-year-old patient is hospitalized with seizures. He has been suffering from a duodenal ulcer for many years. During the last month, he has been vomiting daily, lost weight 20 kg. Objectively: the patient is exhausted. A 'slap noise' is detected in the epigastric region. The lower border of the stomach at the level of the comb line. Laboratory: total protein - 47 g/l, K - 2.1 mmol/l, Na - 118 mmol/l, Ca - 1.6 mmol/l, chlorides - 82 mmol/l, hematocrit - 64%. What is the previous diagnosis?

Декомпенсований стеноз воротаря Decompensated gatekeeper stenosis

Кровоточива виразка 12-ти палої кишки Bleeding duodenal ulcer

Малігнізована виразка шлунка Malignant gastric ulcer

Компенсований стеноз воротаря Compensated stenosis of the goalkeeper

Субкомпенсований стеноз воротаря Subcompensated stenosis of the goalkeeper

78 / 200
Вагітна в терміні 34 тижні доставлена в пологовий будинок в зв’язку з кровотечею зі статевих шляхів, що з’явилася під час сну. Біль не турбує. Кровотеча близько 300,0 мл. Поставте діагноз: A 34-week pregnant woman was taken to the maternity ward due to bleeding from the genital tract that appeared during sleep. The pain does not bother her. The bleeding is close 300.0 ml Make a diagnosis:

Рак шийки матки Cervical cancer

Розрив судин пуповини Rupture of umbilical cord vessels

Передлежання плаценти Placenta previa

Кровоточива ерозія Bleeding erosion

Передчасне відшарування нормально розташованої плаценти Premature detachment of a normally located placenta

79 / 200
Хворий 60-ти років скаржиться на часті болісні утруднені сечовипускання, переривчастий струмінь сечі. Вночі сечовипускання до 5 разів. Після фізичного навантаження відзначає домішки крові у сечі. Ректально простата збільшена, щільно-еластична, безболісна, без вузлів. У сечі: питома вага -1020, білок - 0,1 г/л, лейкоцити - 20-30 у п/з, еритроцити -10-20 у п/з. Який метод обстеження першочерговий? A 60-year-old patient complains of frequent, painful, difficult urination, an intermittent stream of urine. At night, he urinates up to 5 times. After physical exertion, he notes blood impurities in the urine. Rectal prostate enlarged, dense-elastic, painless, without knots. In urine: specific gravity -1020, protein - 0.1 g/l, leukocytes - 20-30 in p/z, erythrocytes -10-20 in p/z. What method is the examination a priority?

Бактеріологічне дослідження сечі Urine bacteriological examination

Трансректальне ультразвукове дослідження Transrectal ultrasound examination

Цистоскопія Cystoscopy

Оглядова урографія Review urography

Екскреторна урографія Excretory urography

80 / 200
Трудова діяльність робітника потребує зосередженості уваги, емоційного напруження. Мають місце зоро- і слухомоторні реакції, висока щільність робочого часу. Дану працю слід оцінювати з точки зору: A worker's work requires concentration of attention, emotional stress. There are visual and auditory-motor reactions, a high density of working time. This work should be evaluated from the point of view of:

Кількості енерговитрат Numbers of energy consumption

Нервового напруження Nervous tension

Статичного навантаження Static load

Фізичної важкості Physical difficulty

Споживання кисню Oxygen consumption

81 / 200
Хвора 48-ми років надійшла до хірургічного відділення з ранами стегна. При огляді встановлено, що поверхня рани вкрита брудно-сірим нальотом з неприємним солодкуватим запахом. Рановий вміст нагадує малинове желе. Шкірні покриви навколо рани напружені, блискучі. Під час пальпації тканин спостерігається помірна крепітація. Яка мікрофлора найбільш імовірно стала причиною запалення? A 48-year-old patient came to the surgical department with thigh wounds. Upon examination, it was found that the surface of the wound was covered with a dirty-gray coating with an unpleasant sweet smell. The wound contents resemble raspberry jelly. The skin around the wound is tight, shiny. During tissue palpation, there is a moderate crepitation. Which microflora most likely caused the inflammation?

Анаеробна клостридіальна Anaerobic clostridial

Анаеробна неклостридіальна Anaerobic non-clostridial

Стафілококи Staphylococci

Синьогнійна паличка Blue pus bacillus

Стрептококи Streptococci

82 / 200
Для обслуговування населення в міській поліклініці є 30 посад дільничних лікарів. Крім цього на території обслуговування поліклініки організовані 3 амбулаторії сімейних лікарів (лікарів загальної практики). Надання якого виду лікувально-профілактичної до- помоги забезпечують вказані лікарі? To serve the population in the city polyclinic, there are 30 positions of district doctors. In addition, 3 dispensaries of family doctors (general practitioners) are organized on the service territory of the polyclinic. What type of treatment is provided -prophylactic help is provided by the specified doctors?

- -

Вторинної Secondary

Третинної Tertiary

Первинної Primary

Першої First

83 / 200
Для обслуговування населення в районній поліклініці ЦРЛ є 25 посад лікарів-спеціалістів. Надання якого виду лікувально-профілактичної допомоги забезпечують вказані лікарі? There are 25 positions of specialist doctors to serve the population in the district polyclinic of the Central Regional Medical Center. What type of medical and preventive care do these doctors provide?

Первинної Primary

Третинної Tertiary

Вторинної Secondary

Першої First

- -

84 / 200
Хвора 53-х років скаржиться на ниючий біль унизу живота, значне його збільшення за останні 5 місяців, схуднення, слабкість. При гінекологічному обстеженні виявлено: шийка матки чиста, матка не збільшена, не-болюча, малорухома. З обох боків визначаються пухлини розміром 10х13 см, з нерівною поверхнею, щільної консистенції, нерухомі. При перкусії живота спостерігається флуктуація. Яким буде попередній діагноз? A 53-year-old patient complains of aching pain in the lower abdomen, its significant increase over the past 5 months, weight loss, weakness. Gynecological examination revealed: the cervix is clean, the uterus is not enlarged, non-painful, immobile. On both sides, tumors measuring 10x13 cm, with an uneven surface, of a dense consistency, are immobile. Fluctuation is observed on percussion of the abdomen. What will be the preliminary diagnosis?

Фіброміома матки Uterine fibromyoma

Тубооваріальна пухлина Tuboovarian tumor

Блукаюча нирка Wandering kidney

Рак яєчників Ovarian cancer

Ендометріоз Endometriosis

85 / 200
Швидкою допомогою доставлена жінка з виниклим після затримки менструації переймоподібним болем в правій здухвинній ділянці, що іррадіює в пряму кишку, кров’янистими виділеннями зі статевих шляхів. Об’єктивно: частота серцевих скорочень - 100/хв., артеріальний тиск - 90/60 мм рт.ст. Шкірні покриви бліді. Живіт болючий при пальпації, позитивний симптом Щоткіна-Блюмберга. При гінекологічному дослідженні - зсуви шийки болісні, праві придатки збільшені, болючі, заднє склепіння нависає, виділення кров’янисті. Поставте попередній діагноз: A woman was delivered by ambulance with cramp-like pain in the right pubic area, radiating into the rectum, blood discharge from the genital tract, which arose after the delay of menstruation. Objectively : heart rate - 100/min., blood pressure - 90/60 mm Hg. Pale skin. Abdomen painful on palpation, positive symptom of Stotkin-Blumberg. During gynecological examination - cervical dislocations are painful, right appendages are enlarged, painful, the posterior vault overhangs, the discharge is bloody. Make a preliminary diagnosis:

Апендицит Appendicitis

Апоплексія правого яєчника Apoplexy of the right ovary

Позаматкова вагітність, що перервалася Interrupted ectopic pregnancy

Аборт в ходу Abortion in progress

Гострий правобічний аднексит Acute right-sided adnexitis

86 / 200
Після народження дитини з масою 4500 г послід відшарувався і виділився самостійно. Пологові шляхи не пошкоджені. Стан породіллі задовільний, пульс - 92/хв., ритмічний. Артеріальний тиск - 100/60 мм рт.ст. Матка скорочується погано, почалася кровотеча. Зроблено зовнішній масаж матки, введені утеротонічні препарати. Крововтрата досягла 350 мл і проводжується. Якою буде тактика лікаря? After the birth of a child with a weight of 4500 g, the litter peeled off and was released on its own. The birth canal was not damaged. The condition of the woman in labor is satisfactory, the heart rate is 92/min., rhythmic. Blood pressure - 100/60 mm Hg. The uterus contracts poorly, bleeding has begun. An external massage of the uterus was performed, uterotonic drugs were administered. Blood loss reached 350 ml and is being carried out. What will the doctor's tactics be?

Затискачi на параметри за Шенкелем-Тіканадзе Shenkel-Tikanadze parameter clamps

Екстирпація матки Uterus extirpation

Ручне обстеження матки та масаж матки на кулаку Manual examination of the uterus and massage of the uterus on the fist

Надпіхвова ампутація матки Supravaginal amputation of the uterus

Тампонада порожнини матки Uterine tamponade

87 / 200
Хворий 38-ми років скаржиться на гнійне виділення із лівої половини носа, утруднене носове дихання, головний біль, важкість в ділянці лівої щоки, підвищення температури тіла до 37,5oC. Хворіє впродовж 6-ти днів. Захворювання пов’язує з переохолодженням. Об’єктивно: болючість при пальпації в ділянці лівої щоки. Слизова оболонка лівої половини носової порожнини гіперемована, набрякла, в середньому носовому ході - гнійна полоска. Який найбільш імовірний діагноз? A 38-year-old patient complains of purulent discharge from the left half of the nose, difficult nasal breathing, headache, heaviness in the area of the left cheek, an increase in body temperature to 37, 5oC. He has been ill for 6 days. The disease is associated with hypothermia. Objectively: pain on palpation in the area of the left cheek. The mucous membrane of the left half of the nasal cavity is hyperemic, swollen, and there is a purulent strip in the middle nasal passage. What is the most likely diagnosis ?

Лівобічний гострий фронтит Left-sided acute frontitis

Викривлення носової перегородки вліво Curvation of the nasal septum to the left

Лівобічний гострий етмоїдит Left-sided acute ethmoiditis

Гострий риніт (нежить) Acute rhinitis (runny nose)

Лівобічний гострий гайморит Left-sided acute sinusitis

88 / 200
Хвора 22-х років скаржиться на підвищення температури до 39oC, біль внизу живота, кров’янисто-гнійні виділення зі статевих шляхів. Із анамнезу: 5 днів тому проведено штучний аборт у 8 тижнів вагітності. При бі-мануальному дослідженні: матка збільшена, м’яка, болюча. Додатки з обох сторін не змінені. Яким буде діагноз? A 22-year-old patient complains of an increase in temperature to 39oC, pain in the lower abdomen, bloody and purulent discharge from the genital tract. From the anamnesis: 5 days ago artificial abortion at 8 weeks of pregnancy. On bi-manual examination: the uterus is enlarged, soft, painful. The appendages on both sides are unchanged. What will be the diagnosis?

Гострий метроендометрит Acute metroendometritis

Перфорація матки Uterine perforation

Пельвіоперитоніт Pelvioperitonitis

Розлитий перитоніт Diffuse peritonitis

Гострий аднексит Acute adnexitis

89 / 200
Хворий 46-ти років серед ночі викликав 'швидку допомогу” з приводу раптового, різкого болю, почервоніння і припухлості першого пальця правої стопи, підвищення температури тіла. Напередодні вживав сухе виноградне вино та жирне м’ясо. Раніше нічим подібним не хворів. Поставте попередній діагноз: A 46-year-old patient called an ambulance in the middle of the night because of sudden, sharp pain, redness and swelling of the first toe of the right foot, and an increase in body temperature. The day before, he used dry grape wine and fatty meat. I have never been sick with anything like that before. Make a preliminary diagnosis:

Деформівний остеоартроз Deformative osteoarthritis

Подагричний артрит Gouty Arthritis

Ревматичний артрит Rheumatoid arthritis

Реактивний артрит Reactive Arthritis

Ревматоїдний артрит Rheumatoid arthritis

90 / 200
Жінка 50-ти років скаржиться на підвищення артеріального тиску, головний біль, нудоту. З анамнезу відомо, що 15 років хворіє на ХОЗЛ. Об’єктивно: почервоніння обличчя, артеріальний тиск -170/120 мм рт.ст. Тони серця підсилені. Пульс - 76/хв., ритмічний. В легенях на тлі жорсткого відтінку везикулярного дихання велика кількість сухих розсіяних хрипів. Який засіб контролю артеріального тиску буде найбільш показаним? A 50-year-old woman complains of increased blood pressure, headache, nausea. It is known from the anamnesis that she has been suffering from COPD for 15 years. Objectively: facial redness , blood pressure -170/120 mm Hg. Heart sounds are increased. Pulse - 76/min., rhythmic. In the lungs, against the background of a harsh shade of vesicular breathing, there are a large number of dry scattered rales. What means of controlling blood pressure will be most indicated?

Каптоприл Captopril

Амлодипін Amlodipine

Фуросемід Furosemide

Пропранолол Propranolol

Клофелін Clofelin

91 / 200
Хворий 43-х років через 2 місяці після перенесеного тонзиліту госпіталізований у терапевтичне відділення зі скаргами на задишку, біль в ділянці серця, запаморочення, серцебиття. Об’єктивно: загальний стан важкий. Частота дихання - 35/хв., пульс - 100/хв., аритмічний, (екстрасистолія). Артеріальний тиск - 145/60 мм рт.ст. Тони серця аритмічні (екстрасистолія) послаблення І тону, систолічний шум над верхівкою. Печінка збільшена. ЕКГ: синусова тахікардія, поодинокі шлуночкові екстрасистоли. ЕхоКГ порожнини серця в межах норми, ФВ- 50%. Який найбільш імовірний діагноз? A 43-year-old patient was hospitalized 2 months after tonsillitis with complaints of shortness of breath, heart pain, dizziness, palpitations. Objectively: general condition is serious. Respiratory rate - 35/min., pulse - 100/min., arrhythmic, (extrasystole). Blood pressure - 145/60 mm Hg. Heart sounds are arrhythmic (extrasystole), weakening of the first sound, systolic murmur over the apex The liver is enlarged. ECG: sinus tachycardia, single ventricular extrasystoles. Echocardiography of the heart cavity is within normal limits, FV-50%. What is the most likely diagnosis?

Синдром Дресслера Dressler Syndrome

Інфекційний перикардит Infectious pericarditis

Міокардитичний кардіофіброз Myocardial cardiofibrosis

Дилатаційна кардіоміопатія Dilated cardiomyopathy

Інфекційний міокардит Infectious myocarditis

92 / 200
Хворий 50-ти років зловживає алкоголем протягом 15-ти років. Після останнього запою не вживає спиртне впродовж 5-ти діб. Ховається під ліжко, намагається чинити опір оточуючим, бачить в них чортів та інопланетян, вважає, що він у пеклі, вірно називає своє прізвище та ім’я, по батькові. Яким буде діагноз у даному випадку? A 50-year-old patient has been abusing alcohol for 15 years. After the last drink, he does not drink alcohol for 5 days. He hides under the bed, tries to resist others , sees devils and aliens in them, believes that he is in hell, correctly names his surname and first name, patronymic. What will be the diagnosis in this case?

Алкогольний делірій Alcoholic delirium

Онейроїдна кататонія Oneiroid catatonia

Алкогольний галюциноз Alcoholic hallucinosis

Сутінкове потьмарення свідомості Twilight obscuration of consciousness

Патологічне сп’яніння Pathological intoxication

93 / 200
У юнака 16-ти років на тлі підвищеної сальності шкіри обличчя, верхньої частини спини, передньої поверхні грудей, множинні комедони, періодично з’являються фолікулярні вузлики, пустули. Суб’єктивних відчуттів немає. Який найбільш імовірний діагноз? A 16-year-old boy has multiple comedones, follicular nodules, and pustules periodically appear against the background of increased sebum on the skin of the face, upper back, front surface of the chest. There are no subjective sensations. What is the most likely diagnosis?

Фурункульоз Furunculosis

Вульгарний сикоз Sycosis Vulgaris

Рожеві вугри Pink Eels

Вторинний сифіліс Secondary syphilis

Звичайні вугри Common acne

94 / 200
У післяопераційного хворого, якому 6 днів тому проведено видалення нижньої частки лівої легені в зв’язку з бактеріальною деструкцією і розвитком пневмотораксу, наросла задишка до 48/хв., PaO2 знизився до 50 мм рт.ст., PaCO2 підвищився до 65 мм рт.ст., з’явився кашель, виділення великої кількості харкотиння, загальний ціаноз шкіри, гіпотонія - 60/20 мм рт.ст. При аускультації правої і лівої легень - маса різнокаліберних хрипів, різко ослаблене дихання. Який об’єм невідкладної допомоги потрібно надати хворому першочергово для ліквідації дихальної недостатності? In a postoperative patient who had the lower lobe of the left lung removed 6 days ago due to bacterial destruction and the development of pneumothorax, shortness of breath increased to 48/min. PaO2 decreased to 50 mm Hg, PaCO2 increased to 65 mm Hg, cough appeared, a large amount of sputum, general cyanosis of the skin, hypotension - 60/20 mm Hg. When auscultating the right and left lungs - a lot of wheezing of various calibers, sharply weakened breathing. What volume of emergency care should be provided to the patient as a priority to eliminate respiratory failure?

Киснева терапія Oxygen therapy

Положення ортопное Position orthopnea

Пункція плевральної порожнини Puncture of the pleural cavity

Провести інтубацію трахеї з ШВЛ Carry out tracheal intubation with ventilator

Провести лаваж трахеобронхiального дерева Perform Tracheobronchial Tree Lavage

95 / 200
Жінка 32-х років доставлена в приймальне відділєння машиною швидкої допомоги. В спекотний день працювала тривалий час на полі, збираючи овочі. Відчула головний біль, запаморочення, загальну слабкість, сухість в роті, але продовжувала працювати. Незабаром 'потемніло в очах” виникли блювання, різкий біль в литкових м’язах. Потім виникло рухове збудження, втрата свідомості. Артеріальний тиск - 170/95 мм рт.ст. Температура - 37,8°C. Сформулюйте найбільш імовірний діагноз: A 32-year-old woman was brought to the reception department by an ambulance. On a hot day, she worked for a long time in the field, picking vegetables. She felt a headache, dizziness, general weakness , dryness in the mouth, but continued to work. Vomiting, sharp pain in the calf muscles soon appeared, 'it became dark in the eyes'. Then there was motor excitement, loss of consciousness. Blood pressure - 170/95 mm Hg. Temperature - 37.8 °C. Formulate the most likely diagnosis:

Нейротоксикоз Neurotoxicosis

Гіпертензивний криз Hypertensive crisis

Гіпертермічний синдром Hyperthermic syndrome

Зомління Grinding

Тепловий удар Heatstroke

96 / 200
Хворий 39-ти років скаржиться на задишку та стискальний біль за грудниною у стані спокою. 10 днів тому переніс грип. Об’єктивно: поза вимушена - сидить, нахиливши тулуб уперед, обличчя здутле, ціанотичне, здуті шийні вени. Межі серця значно розширені в обидві сторони, тони глухі, ритмічні, частота серцевих скорочень - 104/хв., частота дихальних рухів - 28/хв. На ЕКГ: зниження вольтажу зубців, конкордантні зміни сегменту ST. На рентгенограмі: кулеподібна тінь серця, ознаки застою в легенях. В крові: швидкість осідання еритроцитів - 38 мм/год. Поставте діагноз: A 39-year-old patient complains of shortness of breath and squeezing pain behind the sternum at rest. He had the flu 10 days ago. Objectively: the position is forced - he sits, leaning the body is forward, the face is swollen, cyanotic, swollen neck veins. The borders of the heart are significantly expanded on both sides, tones are dull, rhythmic, the heart rate is 104/min, the frequency of respiratory movements is 28/min. On the ECG: a decrease in the voltage of the teeth, concordant changes in the ST segment. On the radiograph: a spherical shadow of the heart, signs of congestion in the lungs. In the blood: the sedimentation rate of erythrocytes is 38 mm/h. Make a diagnosis:

Нестабільна стенокардія Unstable angina

Вірусний міокардит Viral myocarditis

Ревматична гарячка Rheumatic fever

Дилатаційна кардіоміопатія Dilated cardiomyopathy

Ексудативний перикардит Exudative pericarditis

97 / 200
Дівчинка 6-ти років надійшла зі скаргами на підвищення температури тіла до 37,2°C, часті та болісні сечовипускання, які з’явилися після переохолодження. У сечі: сеча каламутна, питома вага - 1012, білок - 0,033°/°°, мікроскопія: лейкоцити - 40-45 в п/з, еритроцити - 8-9 в п/з (свіжі), епітелій плаский: 5-8 в п/з, слиз. Який етіологічний фактор у даному випадку найбільш імовірний? A 6-year-old girl came in with complaints of an increase in body temperature to 37.2°C, frequent and painful urination, which appeared after hypothermia. In the urine : cloudy urine, specific gravity - 1012, protein - 0.033°/°°, microscopy: leukocytes - 40-45 in p/z, erythrocytes - 8-9 in p/z (fresh), squamous epithelium: 5-8 in p /z, mucus. What is the most probable etiological factor in this case?

Klebsiella pneumoniae Klebsiella pneumoniae

Candida albicans Candida albicans

Staphylococcus aureus Staphylococcus aureus

Escherichia coli Escherichia coli

Proteus mirabilis Wonderful Proteus

98 / 200
Хворий 50-ти років в результаті різкого згинання шийного відділу хребта під час дорожньо-транспортної пригоди відчув біль в шиї з іррадіацією до латеральної поверхні правого плеча. Під час обстеження встановлена зона гіпестезії на променевій поверхні правого передпліччя, в ділянці вказівного та середнього пальців, випадіння розгинальноліктьового рефлексу. Вкажіть найбільш імовірний діагноз: A 50-year-old patient experienced pain in the neck radiating to the lateral surface of the right shoulder as a result of sudden bending of the cervical spine during a traffic accident. During the examination a hypoesthesia zone is established on the radial surface of the right forearm, in the area of the index and middle fingers, loss of the elbow extensor reflex. Specify the most likely diagnosis:

Гематомієлія Hematomyelia

Стиснення спинного мозку епідуральною гематомою Spinal cord compression by epidural hematoma

Стиснення спинного мозку відламками хребця Spinal cord compression by vertebral fragments

Забиття спинного мозку Spinal cord injury

Травматичний радикуліт Traumatic sciatica

99 / 200
Хворому 54 роки. Впродовж року лікується з приводу недиференційованої форми раку легень. Наразі приєдналися відчуття тяжкості та болючість у правому підребер’ї. Який метод діагностики буде найбільш інформативним в даному випадку? The patient is 54 years old. He has been treated for an undifferentiated form of lung cancer for a year. Heaviness and pain in the right hypochondrium have now joined. Which method of diagnosis will be most informative in in this case?

Комп’ютерна томографія органів черевної порожнини Computer tomography of abdominal organs

Діагностична лапароскопія Diagnostic laparoscopy

Ультразвукова біолокація з прицільною пункційною біопсією Ultrasound biolocation with targeted puncture biopsy

Радіоізотопна гепатографія Radioisotope hepatography

Вазографія печінки Liver Vasography

100 / 200
В районі N 30% випадків раку молочної залози і 26% випадків раку шийки матки виявляються в IV стадії. Яке управлінське рішення в цій ситуації слід прийняти? In area N, 30% of breast cancer cases and 26% of cervical cancer cases are found in stage IV. What management decision should be taken in this situation?

Організувати і провести комплексні медичні огляди жінок Organize and conduct comprehensive medical examinations of women

Організувати і провести тотальні медичні огляди жінок Organize and conduct total medical examinations of women

Організувати і провести періодичні медичні огляди жінок Organize and conduct periodic medical examinations of women

Організувати і провести цільові медичні огляди жінок Organize and conduct targeted medical examinations of women

Організувати і провести попередні медичні огляди жінок Organize and conduct preliminary medical examinations of women

101 / 200
Управлінню охорони здоров’я необхідно оцінити ефективність впровадження програм з охорони материнства і дитинства в області. Рівень якого інтегрального показника найдоцільніше використати для такого аналізу? The Department of Health Care needs to assess the effectiveness of the implementation of maternity and childhood care programs in the region. Which integral indicator level is the most appropriate to use for such an analysis?

Фізичного розвитку Physical development

Інвалідності дітей Children's disabilities

Захворюваності дітей Children's diseases

Захворюваності жінок Diseases of women

Смертності немовлят Infant Mortality

102 / 200
Хвора 22-х років скаржиться на часте і болюче сечовипускання, поклики на сечо-пуск вночі, нетримання сечі, біль у надлобковій ділянці та у попереку. Часто сеча має колір пива. Місяць тому вийшла заміж. Об’єктивно: загальний стан задовільний. В легенях - везикулярне дихання. Тони серця ритмічні, частота серцевих скорочень - 78/хв., артеріальний тиск - 128/68 мм рт.ст. Живіт м’який, болючий у надлобковій ділянці. У сечі: еритроцити - 12-18 в полі зору, лейкоцити - 12-15, бактерій +++ в полі зору. Який найбільш імовірний діагноз? A 22-year-old patient complains of frequent and painful urination, calls to urinate at night, urinary incontinence, pain in the suprapubic area and in the lower back. The urine often has beer color. Got married a month ago. Objectively: the general condition is satisfactory. In the lungs - vesicular breathing. Heart sounds are rhythmic, heart rate - 78/min., blood pressure - 128/68 mm Hg. Abdomen is soft , painful in the suprapubic area. In the urine: erythrocytes - 12-18 in the field of vision, leukocytes - 12-15, bacteria +++ in the field of vision. What is the most likely diagnosis?

Інфекція нижніх сечових шляхів - цистит Lower urinary tract infection - cystitis

Гонорея Gonorrhea

Інфекція верхніх сечових шляхів - пієлонефрит Upper urinary tract infection - pyelonephritis

Первинний сифіліс Primary syphilis

Сечокам’яна хвороба Urolithiasis

103 / 200
Хвора 50-ти років, яка страждає на хворобу Аддісона (виникла після перенесеного туберкульозу легень у юному віці), під час пожежі в квартирі сильно налякалася і знепритомніла. Хвора бліда, холодна, пульс ниткоподібний, частота серцевих скорочень -120/хв., артеріальний тиск - 60/30 мм рт.ст. Яке ускладнення виникло у хворої? A 50-year-old patient suffering from Addison's disease (occurred after suffering from pulmonary tuberculosis at a young age), was very frightened and fainted during a fire in her apartment. The patient pale, cold, thread-like pulse, heart rate - 120/min., blood pressure - 60/30 mm Hg. What complication did the patient have?

Синдром Морганьї-Адамса-Стокса Morgani-Adams-Stokes Syndrome

Гостра наднирникова недостатність Acute adrenal insufficiency

Гострий інфаркт міокарда Acute myocardial infarction

Тиреотоксичний криз Thyrotoxic crisis

Тампонада серця Cardiac tamponade

104 / 200
На березі моря ви побачили молодого хлопця, який лежав на землі. Шкіра гусяча, різко ціанотична, на губах біла піна. Пульс частий аритмічний, в легенях дихання не вислуховується. Якими будуть перші дії на до-госпітальному етапі? On the seashore, you saw a young guy lying on the ground. Goosebumps, sharply cyanotic skin, white foam on the lips. The pulse is frequent arrhythmic, breathing is not heard in the lungs What will be the first actions at the pre-hospital stage?

Надати положення з піднятими ногами Give position with raised legs

Розтерти і зігріти хворого Rub and warm the patient

Видалити слиз і піну з рота та ротоглотки, провести штучне дихання Remove mucus and foam from the mouth and oropharynx, perform artificial respiration

Залишити в спокої, викликати швидку допомогу Leave alone, call an ambulance

Зробити різкий удар по груднині Make a sharp blow to the sternum

105 / 200
У хворої 50-ти років, яка страждає на гіпертонічну хворобу понад 10 років, на тлі стресу раптово підвищився артеріальний тиск до 200/110 мм рт.ст. Стан супроводжувався тремтінням тіла, головним болем, тахікардією, загальним збудженням, відчуттям жару та сухості в роті. Призначення яких препаратів є найбільш обґрунтованим? A 50-year-old patient who has been suffering from hypertension for more than 10 years had a sudden rise in blood pressure to 200/110 mmHg due to stress. Condition was accompanied by body tremors, headache, tachycardia, general excitement, a feeling of heat and dryness in the mouth. Which drugs are the most justified?

Блокатори рецепторів ангіотензину II Angiotensin II receptor blockers

^-адреноблокатори ^-adrenoblockers

Антагоністи кальцію дигідропіридинові Dihydropyridine calcium antagonists

Сечогінні Diuretics

Антагоністи кальцію недигідропіридинові Non-dihydropyridine calcium antagonists

106 / 200
Робітник заводу з виготовлення цементу, стаж роботи 21 рік, надійшов зі скаргами на виражене свербіння шкіри. Під час огляду на різних ділянках шкіри визначалися поліморфні висипи. Спочатку вони з’явилися на шкірі кистей і пальців рук, передпліччях, обличчі, а за останні півроку розповсюдилися й на інші ділянки. Місцями є елементи везикуляції і попрілості. Під час перебування у відпустці ці прояви зникали. Шкірна проба з хромом дала позитивний результат. Який найбільш імовірний діагноз? A worker at a cement manufacturing plant, 21 years of experience, came in with complaints of severe skin itching. During the examination, polymorphic rashes were identified on different areas of the skin. Initially, they 'appeared on the skin of the hands and fingers, forearms, face, and over the past six months have spread to other areas. In places there are elements of vesiculation and rash. While on vacation, these manifestations disappeared. A skin test with chromium gave a positive result. What is the most likely diagnosis?

Себорейна екзема Seborrheic Eczema

Професійна кропивниця Professional Urticaria

Професійна екзема Occupational eczema

Обмежений нейродерміт Limited neurodermatitis

Мікробна екзема Microbial Eczema

107 / 200
Хворий 34-х років перебуває на ліку- ванні в психіатричній лікарні з приводу загострення шизофренії. Об’єктивно: перебуває в ліжку, рухливо загальмований, контакт відсутній. На запитання не відповідає. Поза одноманітна, пацієнт гіпомімічний, наявні симптом 'хоботка”’ воскова гнучкість м’язів, симптом 'повітряної подушки”’ В такому стані лишається близько тижня. Харчування парентеральне. Визначте наявний синдром розладу рухово-вольової сфери: A 34-year-old patient is being treated in a psychiatric hospital due to an exacerbation of schizophrenia. Objectively: he is in bed, his mobility is inhibited, there is no contact. On the question is not answered. The posture is monotonous, the patient is hypomimic, the symptom 'proboscis' is present, the waxy flexibility of the muscles, the symptom 'air cushion' remains in this state for about a week. Nutrition is parenteral. Determine the present syndrome of motor-volition disorder:

Екзогенний ступор Exogenous Stupor

Апатичний ступор Apathetic Stupor

Кататонічний ступор Catatonic stupor

Депресивний ступор Depressive stupor

Психогенний ступор Psychic Stupor

108 / 200
Хвора 32-х років скаржиться на підвищення температури тіла до 39oC з остудою, біль у поперековій ділянці справа, дизурію протягом доби. На ізотопній ренограмі - праворуч обструктивний тип кривої. Якими мають бути першочергові дії лікаря? A 32-year-old patient complains of an increase in body temperature to 39oC with a chill, pain in the lumbar region on the right, dysuria during the day. On the isotope renogram, there is an obstructive type of curve on the right What should be the primary actions of the doctor?

Введення діуретиків Introduction of diuretics

Антибактеріальна терапія Antibacterial therapy

Відновлення відтоку сечі з правої нирки Restoration of urine outflow from the right kidney

Фізіотерапія для поперекової ділянки Physiotherapy for the lumbar area

Дезінтоксикаційна терапія Detoxification therapy

109 / 200
Забійник вугільної шахти 38-ми років, стаж 15 років, скаржиться на кашель з виділенням слизового харкотиння темного кольору, задуху при фізичному навантаженні, біль у грудях під час дихання. Аускультатив-но: жорстке дихання, сухі та вологі хрипи. На рентгенограмі: бронхо-судинний малюнок посилений, деформований, поодинокі вогнищеві тіні діаметром до 1-3 мм в середніх та нижніх відділах легень, корені розширені, ущільнені. Функція зовнішнього дихання порушена за обструктивним типом. Встановіть попередній діагноз: A 38-year-old coal mine worker, 15 years of experience, complains of a cough with dark-colored sputum, shortness of breath during physical exertion, chest pain when breathing Auscultatively: hard breathing, dry and moist rales. On the radiograph: the bronchovascular pattern is enhanced, deformed, single focal shadows with a diameter of up to 1-3 mm in the middle and lower parts of the lungs, the roots are expanded, compacted. The function of external breathing is impaired according to obstructive type. Establish a preliminary diagnosis:

Хронічне обструктивне захворювання легень Chronic obstructive pulmonary disease

Антракоз Антракоз

Туберкульоз Tuberculosis

Талькоз Талькоз

Бронхопневмонія Bronchopneumonia

110 / 200
Хворий 35-ти років скаржиться на біль у верхній третині плеча, який посилюється вночі. При огляді: помірний набряк у верхній третині плеча, шкіра над ним підвищеної температури, болючість при пальпації, обмеження рухів в плечовому суглобі. На рентгенограмах: ділянка деструкції плечової кістки у метадіафізарному відділі з явищами гомілкового періоститу (спікули) і відшаруванням окістя у вигляді 'дашка”’ Встановіть попередній діагноз: A 35-year-old patient complains of pain in the upper third of the shoulder, which worsens at night. On examination: moderate swelling in the upper third of the shoulder, the skin over it has an elevated temperature, pain during palpation, limitation of movements in the shoulder joint. On X-rays: a site of destruction of the humerus in the metadiaphyseal section with signs of tibial periostitis (spicules) and flaking of the periosteum in the form of a 'dash' Establish a preliminary diagnosis:

Хондробластома Chondroblastoma

Гемангіома Hemangioma

Остеома Osteoma

Хондрома Chondroma

Остеогенна саркома Osteogenic sarcoma

111 / 200
Жінка 53-х років протягом 2-3 місяців скаржиться на постійний ниючий біль в по- переку, відчуває потяги до сечовипускання після фізичного перенавантаження. Візуальних змін сєчі немає. До теперішнього часу захворювань нирок не було. При проведенні ультразвукового дослідження (УЗД) органів черевної порожнини отримані наступні дані: на рівні IV-V поперекових хребців виявлено S-подібне утворення неоднорідної структури, розміром 14х12,5 см. Попередній діагноз: вроджена вада розвитку (S-подібна нирка). Який метод діагностики є найбільш інформативним для підтвердження діагнозу? A 53-year-old woman complains of constant aching pain in the lower back for 2-3 months, feels urges to urinate after physical overload. There are no visual changes in the urine. Until now, there were no kidney diseases. The following data were obtained during an ultrasound examination of the abdominal organs: at the level of the IV-V lumbar vertebrae, an S-shaped formation of a heterogeneous structure, measuring 14x12.5 cm, was found. Preliminary diagnosis: congenital malformation ( S-shaped kidney). Which diagnostic method is the most informative to confirm the diagnosis?

Екскреторна урографія Excretory urography

Радіонуклідна ренографія Radionuclide renography

Ангіографія Angiography

Термографія Thermography

Оглядова рентгенограма нирок Review radiograph of kidneys

112 / 200
Жінка 60-ти років скаржиться на періодичний біль у колінних суглобах, що виникає частіше під час тривалого ходіння, спуску сходами, ввечері, непокоїть у першій половині ночі і вщухає до ранку після тривалого відпочинку. В ході обстеження виявлена надмірна маса тіла. Суглоби зовнішньо не змінені, обсяг рухів в них не обмежений. На рентгенограмі правого колінного суглоба видно остеофіти. Для профілактики подальшого прогресування захворювання слід рекомендувати: A 60-year-old woman complains of periodic pain in the knee joints, which occurs more often during long walks, going down stairs, in the evening, disturbs in the first half of the night and subsides until the morning after a long rest. During the examination, excessive body weight was revealed. The joints are not externally changed, their range of motion is not limited. Osteophytes are visible on the radiograph of the right knee joint. To prevent further progression of the disease, it is recommended:

Уникати підвищених навантажень на колінні суглоби Avoid increased loads on knee joints

Обмежити у дієті продукти, багаті на пурини Limit foods rich in purines in your diet

Короткочасна іммобілізація суглоба лонге-тою Short-term immobilization of the longeto joint

Щоденний біг підтюпцем Daily jogging

Регулярний прийом алопуринолу Regular intake of allopurinol

113 / 200
У повторнороділлі 30-ти років пологи тривають 8 годин. Перейми через кожну хвилину по 50 секунд, активні. Серцебиття плоду - 156/хв., ритмічне. Під час зовнішнього дослідження голівка розташована в порожнині малого тазу. Вагінально: розкриття шийки матки повне, голівка плоду в площині виходу з малого тазу. Стрілоподібний шов в прямому розмірі, мале тім’ячко біля лона. Який це період пологів? In a repeat birth of 30 years, labor lasts 8 hours. Contractions every minute for 50 seconds, active. Fetal heartbeat - 156/min, rhythmic. During external examination, the head is located in the cavity of the small pelvis. Vaginal: opening of the cervix is complete, the fetal head is in the plane of exit from the small pelvis. Arrow-shaped seam in a straight size, small crown near the womb. What is the period of childbirth?

Стрімкі пологи Rapid childbirth

Прелімінарний період Preliminary period

Латентна фаза першого періоду нормальних пологів Latent phase of the first period of normal childbirth

Активна фаза першого періоду нормальних пологів Active phase of the first period of normal childbirth

Другий період нормальних пологів Second period of normal childbirth

114 / 200
Повторновагітна 24-х років з Rh-негативним типом крові перебуває під наглядом перинатолога. В анамнезі: у попередніх пологах проведене ручне відділення плаценти з приводу кровотечі у III періоді. У терміні 36 тижнів вагітності підвищився титр антитіл з 1:16 до 1:64. При УЗД - потовщення плаценти і сповільнення рухів плода. З якою частотою в подальшому необхідно проводити дослідження крові на Rh-антитіла? A 24-year-old re-pregnant woman with Rh-negative blood type is under the supervision of a perinatologist. History: manual separation of the placenta due to bleeding in the III period was performed in previous births. At the 36th week of pregnancy, the titer of antibodies increased from 1:16 to 1:64. On ultrasound, there is thickening of the placenta and slowing of fetal movements. With what frequency in the future is it necessary to conduct a blood test for Rh antibodies?

Перед пологами Before the curtains

1 раз на два тижні Once every two weeks

1 раз на тиждень Once a week

Щоденно до розродження Every day before giving birth

1 раз на три тижні Once every three weeks

115 / 200
У хворого спостерігається відчуження психічних функцій, свого ”Я”: ”я став якимось іншим, відчуваю це, але описати не можу” ”це і я, і не я”’ ”думки мої в тумані, вони не мої” ”мову свою чую десь збоку”’ ”усі мої відчуття пропали”. Визначте психопатологічний синдром: The patient has an alienation of mental functions, his 'I': 'I became something else, I feel it, but I can't describe it' 'it's both me and not me'' 'my thoughts are in a fog, they are not mine' 'I hear my speech somewhere on the side'' 'all my feelings have disappeared'. Define a psychopathological syndrome:

Параноїдний Paranoid

Дереалізації Derealizations

Іпохондричний Hypochondriac

Депресивний Depressed

Деперсоналізації Depersonalizations

116 / 200
25-літній жінці з метою лікування гострого пієлонефриту було призначено 10-денний комплекс антибіотикотерапії. Після закінчення прийому антибіотиків з’явилися густі білі виділення з піхви сирнистого характеру, які супроводжувалися гіперемією зовнішніх статевих органів, вираженим свербінням вульви. Для описаного типу вагініту оберіть раціональну терапію: A 25-year-old woman was prescribed a 10-day complex of antibiotic therapy for the treatment of acute pyelonephritis. After the end of taking antibiotics, thick, white, cheesy vaginal discharge appeared, which were accompanied by hyperemia of the external genitalia, pronounced itching of the vulva. For the described type of vaginitis, choose a rational therapy:

Вагінальні свічки з сульфаніламідами Vaginal suppositories with sulfonamides

Протигрибкові препарати Antifungal drugs

Цефтріаксон Ceftriaxone

Естрогенвмісні креми Estrogen-containing creams

Спринцювання розчином борної кислоти Dousing with boric acid solution

117 / 200
Хворий скаржиться на гнійні виділення з пенісу вранці перед сечовиділенням, різі, полакіурію. Визначена піурія при проведені трьохстаканної проби в першій порції сечі. Поставте діагноз: The patient complains of purulent discharge from the penis in the morning before urinating, cuts, pollakiuria. Pyuria was determined when a three-glass sample was taken in the first portion of urine. Make a diagnosis:

Простатит Prostatitis

Цистит Cystitis

Баланопостит Balanoposthitis

Пієлонефрит Pyelonephritis

Уретрит Уретрит

118 / 200
Дівчинці 13-ти років на підставі клінічних, біохімічних, ЕМГ змін та результату біопсії м’яза поставлено діагноз: первинний ідіопатичний дерматоміозит, первинно-хронічний перебіг, мінімальна активність, ФН 1. Оптимальним вибором лікування буде: A 13-year-old girl was diagnosed on the basis of clinical, biochemical, EMG changes and the result of a muscle biopsy: primary idiopathic dermatomyositis, primary-chronic course, minimal activity , FN 1. The optimal choice of treatment will be:

Гормонотерапія Hormone therapy

Нестероїдні протизапальні препарати Nonsteroidal anti-inflammatory drugs

Цитостатична терапія Cytostatic therapy

Масаж, ЛФК Massage, exercise therapy

Бальнеотерапія Balneotherapy

119 / 200
Жінка 72-х років хворіє на цукровий діабет II типу, супутня патологія - гіпертонічна хвороба II ст., серцева недостатність II Б ст. Використовує метформін. Напередодні перенесла гіпертонічний криз після чого з’явилися різка слабкість, міалгії, збільшилася спрага, сухість у роті, поліурія. Артеріальний тиск - 140/95 мм рт.ст., частота серцевих скорочень - 98/хв., набряки та запах ацетону відсутні. Які заходи слід вжити для хворої, щоб попередити розвиток коматозного стану? A 72-year-old woman suffers from type II diabetes, concomitant pathology - hypertension of the II stage, heart failure of the II B stage. She uses metformin. The day before, she had a hypertensive crisis followed by sharp weakness, myalgias, increased thirst, dry mouth, polyuria. Blood pressure - 140/95 mm Hg, heart rate - 98/min, swelling and acetone smell are absent. What measures should be taken take for the patient to prevent the development of a comatose state?

Призначення глібенкламіду Prescribing Glibenclamide

Збільшення дози метформіну в два рази Metformin dose doubled

Використання гіпотонічного розчину хлориду натрію Use of hypotonic sodium chloride solution

Додаткове призначення пролонгованого інсуліну Additional assignment of extended-release insulin

Відміна метформіну, призначення інсуліну короткої дії Withdrawal of metformin, appointment of short-acting insulin

120 / 200
Хвора 54-х років скаржиться на мерзлякуватість, сонливість, слабкість. В минулому лікувалася з приводу аутоімунного вузлового зоба. За рік її стан погіршився: з’явилася слабкість, набрала вагу, стала повільною. Щитоподібна залоза -1 ст. Шкіра холодна, суха, бліда, язик потовщений. Голос захриплий. Тони серця ритмічні, глухі, частота серцевих скорочень - 56/хв., живіт здутий. Випорожнення - закрепи. Який препарат слід призначити для профілактики значного збільшення щитоподібної залози? A 54-year-old patient complains of chills, drowsiness, weakness. In the past, she was treated for autoimmune nodular goiter. Over the course of a year, her condition worsened: weakness appeared, gained weight, became sluggish. Thyroid gland - 1 st. Skin cold, dry, pale, tongue thickened. Voice hoarse. Heart sounds rhythmic, dull, heart rate - 56/min., abdomen distended. Stools - constipation. What drug should be taken appoint for the prevention of a significant increase in the thyroid gland?

Метилтіоурацил Methylthiouracil

Преднізолон Prednisone

Тироксин Tyroxin

Радіоактивний йод Radioactive iodine

Мерказоліл Mercazolil

121 / 200
Дитина 6-ти років з наявністю анемічного та геморагічного синдромів. В крові: НЬ-80 г/л, КП- 0,9, ретикулоцити - 2o/oo, лейкоцити - 1,0109/л, тромбоцити - 10109/л. Який діагноз є найбільш імовірним? A 6-year-old child with anemic and hemorrhagic syndromes. In the blood: Hb-80 g/l, CP- 0.9, reticulocytes - 2o/oo , leukocytes - 1.0109/l, platelets - 10109/l. What diagnosis is the most probable?

Тромбоцитопатія Thrombocytopathy

Апластична анемія Aplastic anemia

Лімфобластний лейкоз Lymphoblastic leukemia

Дефіцитна анемія Deficiency anemia

Тромбоцитопенічна пурпура Thrombocytopenic purpura

122 / 200
Пацієнт 60-ти років скаржиться на практично постійне відчуття важкості та переповнення в епігастрії, що посилюється після їжі, відрижку з тухлим запахом, іноді блювання з’їденою 1-2 дні тому їжею, схуднення. 12 років тому вперше виявлена виразка пі-лоричного каналу. Спостерігав періодичний 'голодний” біль, з приводу якого приймав ранітидин. Погіршення протягом 3-х місяців. Об’єктивно: визначається ”шум плескоту” в епігастрії. Про яке ускладнення йдеться? A 60-year-old patient complains of an almost constant feeling of heaviness and fullness in the epigastrium, which worsens after eating, belching with a rotten smell, sometimes vomiting eaten 1- 2 days ago I eat, weight loss. Pyloric canal ulcer was first detected 12 years ago. Observed periodic 'hungry' pain, for which he took ranitidine. Worsening during 3 months. Objectively: a 'cluttering noise' is detected in the epigastrium. What complication is it about?

Пенетрація виразки шлунка Stomach ulcer penetration

Малігнізація виразки шлунка Malignancy of gastric ulcer

Стороннє тіло шлунка (безоар) Foreign body of the stomach (bezoar)

Функціональний спазм воротаря Functional spasm of the goalkeeper

Стеноз пілоруса Stenosis of the pylorus

123 / 200
Хворий 25-ти років надійшов через 2 години після дорожньо-транспортної аварії зі скаргами на постійний інтенсивний біль у правій паховій ділянці та у симфізі тазу, неможливість самостійно пересуватися. При огляді: положення хворого вимушене - поза ”жаби” (позитивний симптом Волковича). Садна й синці на шкірі в проекції симфізу та горизонтальної гілки лобкової кістки праворуч, виражений набряк м’яких тканин. Позитивний симптом ’’прилиплої п’яти” праворуч. Поставте попередній діагноз: A 25-year-old patient arrived 2 hours after a traffic accident with complaints of constant intense pain in the right inguinal area and pelvic symphysis, inability to move independently. During the examination: the patient's position is forced - the 'frog' position (positive Volkovych's symptom). Bruises and bruises on the skin in the projection of the symphysis and the horizontal branch of the pubic bone on the right, pronounced swelling of soft tissues. Positive symptom of 'sticky heel' on the right. Make a preliminary diagnosis:

Закритий перелом лобкової кістки праворуч Closed fracture of pubic bone on the right

Травматична аневризма стегнової артерії Traumatic femoral artery aneurysm

Вивих правого стегна Dislocation of right hip

Забій кісток тазу Pelvic fracture

Травматична пахвинна грижа Traumatic Groin Care

124 / 200
Хворого після перенесеного Q-інфар-кту міокарда турбує задишка при незначних навантаженнях, нічні напади сухого кашлю, відчуття хрипів у грудній клітці і ортопное. В ході огляду: акроціаноз, частота серцевих скорочень - 96/хв., тахікардія, ослаблення Iго тону над верхівкою, вислуховується III-й тон. В легенях в базальних відділах незвучні хрипи. Під час ехокардіоскопії дилатація лівого шлуночка, витончення і дискінезія міжшлуночкової перегородки, фракція викиду -39%. Визначити варіант дисфункції міокарда: After a Q-myocardial infarction, the patient is troubled by shortness of breath with slight exertion, night attacks of dry cough, a feeling of wheezing in the chest and orthopnea. During the examination: acrocyanosis , heart rate - 96/min., tachycardia, weakening of the 1st tone above the apex, the 3rd tone is heard. In the lungs in the basal parts, there are silent rales. During echocardioscopy, dilatation of the left ventricle, thinning and dyskinesia of the interventricular septum, ejection fraction -39% Determine the variant of myocardial dysfunction:

Діастолічна дисфункція правого шлуночка Diastolic dysfunction of the right ventricle

Діастолічна дисфункція лівого шлуночка Diastolic dysfunction of the left ventricle

Систолічна дисфункція лівого шлуночка Left ventricular systolic dysfunction

Систолічна дисфункція правого шлуночка Right ventricular systolic dysfunction

Змішана дисфункція обох шлуночків Mixed dysfunction of both ventricles

125 / 200
Дівчинка 7-ми років скаржиться на біль в ділянці піхви, значні виділення гнійного характеру, які турбують протягом 5-ти днів і поступово посилюються. При огляді лікар виявив значний набряк зовнішніх статевих органів, почервоніння, гнійні виділення з піхви з неприємним запахом. При ультразвуковому дослідженні (УЗД) в ділянці піхви визначається ехо-позитивна тінь. Яка причина може призвести до такого стану у дітей? A 7-year-old girl complains of pain in the vagina, significant discharge of a purulent nature, which bothers her for 5 days and gradually worsens. During the examination, the doctor found a significant swelling of the external genitalia, redness, purulent discharge from the vagina with an unpleasant smell. An echo-positive shadow is determined in the vaginal area during ultrasound examination. What cause can lead to such a condition in children?

Вульвовагініт Vulvovaginitis

Пухлина піхви Vaginal tumor

Гнійний кольпіт Purulent colpitis

Наявність стороннього тіла в піхві The presence of a foreign body in the vagina

Пухлина шийки матки Cervical tumor

126 / 200
Дитина 1,5 років у розпал гострої респіраторної вірусної інфекції (ГРВ!) їла волоський горіх. Раптово закашлялася, посиніла. Періодичний сухий кашель зі свистом на видиху продовжується. Стан дитини легкого ступеню важкості, дихальна недостатність. Вкорочення перкуторного звуку, жорстке дихання, сухі і вологі хрипи, бронхоспазм на видиху над правою легенею. Якою буде подальша тактика: A 1.5-year-old child ate a walnut in the midst of an acute respiratory viral infection (ARV). He suddenly coughed and turned blue. The periodic dry cough with a whistling sound continues. The child's condition is mild, respiratory insufficiency. Shortening of the percussion sound, hard breathing, dry and moist wheezing, bronchospasm on exhalation over the right lung. What will be the further tactics:

Оглядова рентгенографія живота Comprehensive radiography of the abdomen

Оглядова рентгенографія грудної клітки Comprehensive chest X-ray

Бронхоскопія Bronchoscopy

Ультразвукове дослідження (УЗД) живота та грудної клітки Ultrasound examination (ultrasound) of abdomen and chest

Комп’ютерна томограма грудної клітки Computer tomography of the chest

127 / 200
До дільничного терапевта звернувся пацієнт 43-х років зі скаргами на печію, відрижку кислим, появу болю під час ковтання їжі, а також неможливістю вживати тверду їжу. Хворий повідомив, що печія та відрижка кислим у нього спостерігається впродовж 8-ми років. Погіршення свого стану спостерігає останні 3 місяці. Яке обстеження потрібно пройти пацієнту в першу чергу? A 43-year-old patient turned to the district therapist with complaints of heartburn, acid belching, pain when swallowing food, and the inability to eat solid food. The patient reported , that he has had heartburn and acid belching for 8 years. He has been observing a worsening of his condition for the past 3 months. What examination should the patient undergo first of all?

Багатогодинний езофаго-рН-моніторинг Multi-hour esophageal pH monitoring

Фіброезофагогастродуоденоскопія Fibroesophagogastroduodenoscopy

Комп’ютерна томографія органів грудної клітки Computer tomography of chest organs

Добовий езофаго-рН-моніторинг Daily esophageal pH monitoring

Тест з інгібітором протонової помпи Proton pump inhibitor test

128 / 200
У хлопчика 5-ти років після перенесеної ангіни на 5-й день з’явився біль у правому кульшовому суглобі. Температура тіла до 39oC. Лейкоцитоз - 18, 0 • 109/л, зсув формули вліво. Рухи в кульшовому суглобі болючі, обмежені. Живіт м’який, неболючий. На рентгенограмах кісток таза виявлено зону деструкції. При ретельному дослідженні знайдено інфільтрацію в ділянці правої здухвинної кістки, болюча, напружена. Поставте попередній діагноз: A 5-year-old boy developed pain in the right hip joint on the 5th day after a sore throat. Body temperature up to 39oC. Leukocytosis - 18.0 • 109/l, shift of the formula to the left. Movements in the hip joint are painful, limited. The abdomen is soft, painless. X-rays of the pelvic bones revealed a zone of destruction. Upon careful examination, infiltration was found in the area of the right iliac bone, painful, tense. Make a preliminary diagnosis :

Токсико-алергічний коксит Toxic-allergic coxitis

Доброякісна пухлина правої здухвинної кістки Benign tumor of the right iliac bone

Гострий гематогенний остеомієліт правої здухвинної кістки Acute hematogenous osteomyelitis of the right iliac bone

Злоякісна пухлина правої здухвинної кістки Malignant tumor of the right iliac bone

Іострий апендицит Acute appendicitis

129 / 200
Хворий 29-ти років на другий день після приїзду з Індії госпіталізований у клініку з скаргами на біль у животі, рідкі випорожнення зі склоподібним слизом та кров’ю у вигляді 'малинового желе’.’ Загальний стан задовільний, шкіра бліда, висипу немає. Живіт м’який, чутливий в ділянці сліпої і висхідної кишок. Оберіть етіотропний препарат для лікування цієї патології: On the second day after arriving from India, a 29-year-old patient was hospitalized in the clinic with complaints of abdominal pain, loose stools with glassy mucus and blood in the form of 'raspberry jelly'.' The general condition is satisfactory, the skin is pale, there is no rash. The abdomen is soft, sensitive in the area of the cecum and ascending colon. Choose an etiotropic drug for the treatment of this pathology:

Цефтріаксон Ceftriaxone

Метронідазол Metronidazole

Дексазон Dexazon

Еритроміцин Erythromycin

Ніфуроксазид Nifuroxazide

130 / 200
43-річний чоловік без постійного місця проживання госпіталізований на 5-й день захворювання зі скаргами на підвищення температури тіла до 39,6oC, загальну слабкість, головний біль. Об’єктивно: збуджений, ей-форичний. На одязі хворого велика кількість вошей. Обличчя гіперемоване, набрякле. Одиничні петехії на перехідній складці кон’юнктиви, на шкірі розеольозно-петехіальна висипка. Язик при висовуванні тремтить та відхиляється вліво. Збільшені печінка та селезінка. Який найбільш імовірний діагноз? A 43-year-old man without a permanent place of residence was hospitalized on the 5th day of illness with complaints of an increase in body temperature to 39.6oC, general weakness, headache. About Objectively: excited, euphoric. There are a large number of lice on the patient's clothes. The face is hyperemic, swollen. Single petechiae on the transitional fold of the conjunctiva, a roseolous-petechial rash on the skin. The tongue trembles when protruding and deviates to the left. The liver and spleen are enlarged. What is the most likely diagnosis?

Інфекційний мононуклеоз Infectious mononucleosis

Висипний тиф Typhoid

Грип Грип

Аденовірусна інфекція Adenovirus infection

Черевний тиф Typhoid

131 / 200
Хворий 68-ми років звернувся до урологічної клініки зі скаргами на поклики до сечопуску, відсутність сечі протягом останніх 10 годин. Об’єктивно: над лобком пальпується болючий утвір, верхній край якого сягає пупка, в ході ректального дослідження передміхурова залоза збільшена, безболісна, щільної консистенції. Додаткове дослідження виявило рівень простатспецифічного антигену в крові - 3,2 нг/мл. Який найбільш імовірний діагноз? A 68-year-old patient came to the urology clinic with complaints of urges to urinate, no urine for the past 10 hours. Objectively: a painful lump is palpated above the pubis, the upper edge of which reaches the navel, during a rectal examination, the prostate gland is enlarged, painless, with a dense consistency. An additional examination revealed the level of prostate-specific antigen in the blood - 3.2 ng/ml. What is the most likely diagnosis?

Хронічний парапроктит, гостра затримка сечі Chronic paraproctitis, acute urinary retention

Склероз передміхурової залози, гостра затримка сечі Sclerosis of the prostate, acute urinary retention

Рак передміхурової залози, гостра затримка сечі Prostate cancer, acute urinary retention

Доброякісна гіперплазія передміхурової залози, хронічна затримка сечі Benign prostatic hyperplasia, chronic urinary retention

Доброякісна гіперплазія передміхурової залози, гостра затримка сечі Benign prostatic hyperplasia, acute urinary retention

132 / 200
Пацієнт 30-ти років після дорожньо-транспортної пригоди (ДТП) непритомний, шкірні покриви бліді, пульс ниткоподібний. В с/3 правого стегна наявна велика рвана рана з рясною тривалою зовнішньою артеріальною кровотечею. Накладено джгут. Кровотеча зупинена. На який максимальний час може бути накладений джгут на кінцівку в літню пору? A 30-year-old patient is unconscious after a traffic accident (traffic accident), the skin is pale, the pulse is filiform. There is a large lacerated wound with with profuse long-term external arterial bleeding. A tourniquet is applied. Bleeding is stopped. What is the maximum time a tourniquet can be applied to a limb in the summer?

Не більше 5 годин No more than 5 hours

Не більше 10 годин No more than 10 hours

Не більше 2 хвилин No more than 2 minutes

Не більше 10 секунд No more than 10 seconds

Не більше 2 годин No more than 2 hours

133 / 200
Жінці 40-ка років 5 років тому проведено двосторонню аднексектомію. Скаржиться на слабкість, втомлюваність, відсутність менструації, біль у попереку, зниження пам’яті, не пам’ятає нещодавні події. При обстеженні наявність ожиріння, остеопорозу та гіперхолестеринемії. Який синдром спостерігається у пацієнтки? A 40-year-old woman underwent a bilateral adnexectomy 5 years ago. She complains of weakness, fatigue, lack of menstruation, lower back pain, decreased memory, no describes recent events. On examination, the presence of obesity, osteoporosis, and hypercholesterolemia. What syndrome is observed in the patient?

Психоневротичний синдром Psychiatric Syndrome

Адіпозогенітальна дистрофія Adiposogenital dystrophy

Посткастраційний синдром Post-castration syndrome

Клімактеричний синдром Climacteric syndrome

Андреногенітальний синдром Andrenogenital syndrome

134 / 200
До клініки надійшов хворий 18-ти років зі скаргами на виражену задишку, кашель, свербіння шкіри. Захворів гостро, близько години тому після прийому анальгіну. З дитинства - кропивниця, ”аспіринова астма” Об’єктивно: параорбітальний набряк, почервоніння та набряк обличчя. Дихання свистяче. Частота пульсу - 96/хв., артеріальний тиск - 80/50 мм рт.ст. Препаратом невідкладної допомоги буде: An 18-year-old patient came to the clinic with complaints of severe shortness of breath, cough, itchy skin. He became acutely ill about an hour ago after taking analgin. He has had urticaria since childhood , 'aspirin asthma' Objectively: paraorbital edema, redness and swelling of the face. Breathing is wheezing. Pulse rate - 96/min., blood pressure - 80/50 mm Hg. Emergency medicine will be:

Супрастин Suprastin

Преднізолон Prednisone

Строфантин Strophanthin

Еуфілін Euphilin

Дофамін Dopamine

135 / 200
У породіллі через 3 тижні після пологів підвищилася температура тіла до 38oC, з’явилися остуда, слабкість та біль в лівій молочній залозі. Молочна залоза збільшена, на-грубла, болюча при пальпації; розм’якшення та флуктуації в ділянці інфільтрату немає. В крові: помірний лейкоцитоз. Який найбільш імовірний діагноз? 3 weeks after giving birth, the mother's body temperature rose to 38oC, chills, weakness and pain in the left mammary gland appeared. The mammary gland is enlarged, rough , painful on palpation; there is no softening and fluctuation in the area of the infiltrate. In the blood: moderate leukocytosis. What is the most likely diagnosis?

Серозний мастит Serous mastitis

Мастопатія Mastopathy

Гангренозний мастит Gangrenous Mastitis

Абсцедивний мастит Abscessive mastitis

Лактостаз Lactic Stasis

136 / 200
Хвора 25-ти років скаржиться на неплідність та вторинну аменорею. Об’єктивно: надмірного харчування, явища гірсутизму. Бімануально: матка нормальних розмірів, яєчники з обох боків дещо збільшені, неболю-чі. Рівень ЛГ та тестостерону підвищений, проба з АКТГ - негативна. Поставте діагноз: A 25-year-old patient complains of infertility and secondary amenorrhea. Objectively: excessive nutrition, hirsutism. Bimanually: uterus of normal size, ovaries on both sides slightly increased, not painful. The level of LH and testosterone is increased, the ACTH test is negative. Make a diagnosis:

Вірилізуючі пухлини яєчників Virilizing ovarian tumors

Синдром полікістозних яєчників Polycystic ovary syndrome

Адреногенітальний синдром Adrenogenital syndrome

Синдром резистентних яєчників Resistant Ovary Syndrome

Двобічний хронічний сальпінгоофорит Bilateral chronic salpingo-oophoritis

137 / 200
Дитині 3-х років у зв’язку із захворюванням на гостру респіраторну вірусну інфекцію (ГРВІ) призначено: бісептол, парацетамол, назоферон. На третій день стан дитини погіршився: з’явилися біль у горлі, стоматит, кон’юнктивіт, гіперсалівація, болючі плями темно-червоного кольору на шиї, обличчі, грудях та кінцівках, потім на місці плям з’явились пухирі. Спостерігалося ураження слизових оболонок навколо рота та ануса. Який попередній діагноз? A 3-year-old child was prescribed: Biseptol, Paracetamol, Nasoferon due to an acute respiratory viral infection (ARSI). On the third day, the child's condition worsened : sore throat, stomatitis, conjunctivitis, hypersalivation, painful spots of dark red color appeared on the neck, face, chest and limbs, then blisters appeared at the spot of the spots. There was damage to the mucous membranes around the mouth and anus. What previous diagnosis?

Синдром Стівенса-Джонсона Stevens-Johnson syndrome

Вітряна віспа Chicken Pox

Бульозний дерматит Bulusous Dermatitis

Сироваткова хвороба Serum sickness

Атопічний дерматит Atopic dermatitis

138 / 200
Емоційнолабільна 22-річна студентка медичного університету присутня під час хірургічної операції відчула слабкість, запаморочення, легку нудоту, втратила свідомість і впала. Такий стан тривав 2 хвилини. Об’єктивно: очі закриті, шкіра бліда, дифузний гіпергідроз, пульс слабкий, артеріальний тиск - 90/60 мм рт.ст., дихання поверхневе, реакція на світло уповільнена, сухожилкові рефлекси збережені, судом не має. Який найбільш імовірний діагноз? An emotionally unstable 22-year-old medical university student present during a surgical operation felt weak, dizzy, mild nausea, lost consciousness and fell. This state lasted for 2 minutes. About' Objectively: eyes are closed, skin is pale, diffuse hyperhidrosis, pulse is weak, blood pressure is 90/60 mm Hg, breathing is shallow, reaction to light is slowed down, tendon reflexes are preserved, there is no seizure. What is the most likely diagnosis?

Транзиторна ішемічна атака Transient ischemic attack

Істеричний невроз Hysterical neurosis

Вегето-судинний пароксизм Vegeto-vascular paroxysm

Епілептичний напад Epileptic attack

Синкопальний стан Syncopal state

139 / 200
В місті N проводилося вивчення захворюваності на інфаркт міокарда за попередні роки (з 2009 по 2013 роки). Який вид епідеміологічного дослідження був використаний? In the city N, the incidence of myocardial infarction was studied in previous years (from 2009 to 2013). What type of epidemiological study was used?

Ретроспективний Retrospective

Проспективний Prospective

Експериментальний Experimental

Аналітичний Analytic

Описовий Descriptive

140 / 200
У хворого 19-ти років проникаюче по- ранення лівої половини грудної порожнини. Стан тяжкий, ціаноз, задишка. Об’єктивно: зліва - дихання різко послаблене. Серце - тони глухі, пульс - 102/хв., артеріальний тиск -90/50 мм рт.ст. Ехокардіографія - виявлений ехонегативний простір, який оточує камеру лівого шлуночка. Невелика кількість рідини у лівій плевральній порожнині та велика кількість повітря. Якому патологічному стану найбільш імовірно відповідає ця картина? A 19-year-old patient has a penetrating wound of the left half of the chest cavity. The condition is severe, cyanosis, shortness of breath. Objectively: on the left - breathing is sharply weakened. Heart - dull tones, pulse - 102/min, blood pressure -90/50 mm Hg. Echocardiography - an echonegative space surrounding the chamber of the left ventricle was detected. A small amount of fluid in the left pleural cavity and a large amount of air. Which pathological condition is most is this picture likely to match?

Дилатаційна кардіоміопатія Dilated cardiomyopathy

Лівобічний гідропневмоторакс та гідропе-рикард Left-sided hydropneumothorax and hydropericardium

Правобічний пневмоторакс та гемоперикард Right-sided pneumothorax and hemopericardium

Лівобічний пневмоторакс та ексудативний перикардит Left-sided pneumothorax and exudative pericarditis

Лівобічний пневмоторакс та гемоперикард Left-sided pneumothorax and hemopericardium

141 / 200
Пацієнтка 35-ти років скаржиться на біль і збільшення правої молочної залози. Страждає на безпліддя 15 років. Права молочна залоза збільшена у розмірах, шкіра її пастозна, гіперемована, пальпується інфільтрат тістоподібної консистенції без чітких контурів, симптом 'лимонної кірки’.’ Поставте діагноз: A 35-year-old patient complains of pain and enlargement of the right mammary gland. She has suffered from infertility for 15 years. The right mammary gland is enlarged, her skin is pasty, hyperemic, palpable infiltrate of a dough-like consistency without clear contours, a symptom of 'lemon peel'. Make a diagnosis:

Абсцес молочної залози Breast abscess

Рак молочної залози Breast cancer

Вузлова мастопатія Nodular mastopathy

Фіброаденома молочної залози Breast fibroadenoma

Мастит Mastitis

142 / 200
Хлопчик 15-ти років протягом останніх 5 років хворіє на виразкову хворобу 12-палої кишки. Останнє загострення хвороби було весною. Раптово з’явився ”кинджальни-й’ біль в епігастрії, який дещо тамувався в колінно-ліктьовому положенні. Стан дитини тяжкий, різка блідість, тахікардія, симптоми подразнення очеревини. Який найбільш імовірний діагноз? A 15-year-old boy has been suffering from ulcer disease of the duodenum for the past 5 years. The last exacerbation of the disease was in the spring. Suddenly, 'dagger-y' appeared pain in the epigastrium, which was somewhat suppressed in the knee-elbow position. The child's condition is severe, sharp pallor, tachycardia, symptoms of peritoneal irritation. What is the most likely diagnosis?

Гострий апендицит Acute appendicitis

Перфорація 12-палої кишки Perforation of the duodenum

Гострий панкреатит Hostry Pancreatitis

Гостра кишкова непрохідність Acute intestinal obstruction

Калькульозний холецистит Calculous Cholecystitis

143 / 200
Хворому 27-ми років з двостороннім гідротораксом неодноразово проводили плевральні пункції з обох сторін. Після чергової пункції - погіршення стану, лихоманка, біль у грудях. Наступного дня під час плевральної пункції справа одержано гній. Встановлено діагноз: правостороння гостра емпієма плеври. Який механізм виникнення даного ускладнення? A 27-year-old patient with bilateral hydrothorax underwent multiple pleural punctures on both sides. After another puncture, the condition worsened, fever, chest pain. The next day during pus was obtained from a pleural puncture on the right. The diagnosis was established: right-sided acute empyema of the pleura. What is the mechanism of this complication?

Лімфогенний Lymphogenic

Повітряний Air

Імплантаційний Implantation

Контактно-аспіраційний Contact-aspiration

Гематогенний Hematogenous

144 / 200
Хлопчикові 14 років. Протягом 3-х років спостерігається підвищення артеріального тиску до 170/100 мм рт.ст. Лікування анти-гіпертензивними препаратами неефективне. ХолтерАТ: стабільна артеріальна гіпертен- зія. Доплер судин нирок: часткова візуаліза-ція ниркових артерiй злiва. Яким буде попередню діагноз? The boy is 14 years old. For 3 years, an increase in blood pressure has been observed up to 170/100 mm Hg. Treatment with anti-hypertensive drugs is ineffective. Holter BP: stable arterial hypertension. Doppler of kidney vessels: partial visualization of the renal arteries on the left. What will be the preliminary diagnosis?

Первинна артеріальна гіпертензія Primary arterial hypertension

Феохромоцитома Pheochromocytoma

Реноваскулярна гіпєртєнзія Renovascular hypertension

Адреногенітальний синдром Adrenogenital syndrome

Гіпоталамічний синдром Hypothalamic syndrome

145 / 200
Для створення безпечних умов праці персоналу у процедурній відділення дистанційної променевої терапії, де з лікувальною метою використовується гамма-терапевтична установка, необхідно застосувати принцип захисту екрануванням. З якого матеріалу повинні бути захисні екрани? To create safe working conditions for personnel in the procedural department of remote radiation therapy, where a gamma therapy unit is used for therapeutic purposes, it is necessary to apply the principle of shielding protection. From what material should be protective screens?

Дерево, цегла Wood, brick

Свинець, баритобетон Lead, barite concrete

Алюміній, парафін Aluminum, Paraffin

Скло, органічне скло Glass, organic glass

Пластмаси, гуми Plastics, tires

146 / 200
Дівчина 16-ти років з метою схуднення постійно після вживання їжі викликає у себе блювання. У психічному статусі: астенізова-на, дратівлива, схильна до істеричних реакцій, агресивна до матері. Об’єктивно: худа, шкіра бліда, підвищене оволосіння тіла, виразка шлунка, аменорея. Визначте психічний розлад: A 16-year-old girl in order to lose weight constantly vomits after eating. Mental status: asthenic, irritable, prone to hysterical reactions, aggressive to the mother. Objectively: thin, pale skin, increased body hair, gastric ulcer, amenorrhea. Define mental disorder:

Фобічний розлад Phobic Disorder

Істеричний розлад Hysterical disorder

Психічна анорексія Anorexia nervosa

Дисфоричний розлад Dysphoric disorder

Депресивний розлад Depressive disorder

147 / 200
Хворий 35-ти років скаржиться на наявність впродовж тривалого часу збільшених периферичних лімфатичних вузлів, які його не турбують. Із анамнезу захворювання: спочатку збільшувалися лімфовузли шиї, надключичної, пахвової ділянки, з’являлися нові групи лімфовузлів. Об’єктивно: лімфовузли при пальпації м’якоеластичної консистенції, збільшені, безболісні, не спаяні з навколишніми тканинами. Який метод дослідження є найбільш інформативним для ранньої діагностики захворювання? A 35-year-old patient complains of the presence of enlarged peripheral lymph nodes for a long time, which do not bother him. From the anamnesis of the disease: initially the lymph nodes of the neck, supraclavicular, axillary areas, new groups of lymph nodes appeared. Objectively: lymph nodes on palpation of a soft-elastic consistency, enlarged, painless, not fused with the surrounding tissues. Which research method is the most informative for early diagnosis of the disease?

Магнітнорезонансна томографія Magnetic resonance imaging

Рентгенологічне дослідження X-ray examination

Пункційна біопсія Puncture biopsy

Ультразвукове обстеження Ultrasound examination

Радіоізотопне сканування скелета Radioisotope scan of the skeleton

148 / 200
Хвора 23-х років звернулася до клініки зі скаргами на серцебиття, пітливість, слабкість, зниження працездатності, тремтіння пальців рук. За місяць схудла на 10 кг. При огляді: частота серцевих скорочень - 120/хв., артеріальний тиск - 130/70 мм рт.ст. Під час ультразвукового дослідження (УЗД) в щитоподібній залозі виявлено вузол 2х2 см. При скануванні в місці вузла виявлена ділянка підвищеного накопичення радіофармпрепарату. Решта тканин щитоподібної залози РФП не накопичує. Який найбільш імовірний діагноз? A 23-year-old patient came to the clinic with complaints of heart palpitations, sweating, weakness, reduced work capacity, trembling of the fingers. She lost 10 kg in a month. During the examination : heart rate - 120/min., blood pressure - 130/70 mm Hg. During an ultrasound examination (USG), a 2x2 cm nodule was detected in the thyroid gland. During the scan at the site of the nodule, an area of increased accumulation of the radiopharmaceutical was detected. The rest of the thyroid tissue the gland does not accumulate RFP. What is the most likely diagnosis?

Змішаний токсичний зоб Mixed toxic goiter

Токсична аденома щитоподібної залози Toxic thyroid adenoma

Тиреотропінома Thyrotropinoma

Підгострий тиреоїдит Subacute thyroiditis

Аутоімунний тиреоїдит Autoimmune thyroiditis

149 / 200
Хворий 22-х років звернувся до лікаря зі скаргами на появу білих плям на тлі засмаглої шкіри. На шкірі тулуба множинні гіпопігментовані плями, при пошкрябуванні спостерігається муковидне лущення. Проба Бальцера позитивна. Яке захворювання можна припустити? A 22-year-old patient turned to the doctor with complaints about the appearance of white spots on the background of tanned skin. There are multiple hypopigmented spots on the skin of the body, when scraping, there is a powdery crust. Sample Balzera is positive. What disease can be assumed?

Склеродермія Scleroderma

Рожевий лишай Pink lichen

Вітіліго Vitiligo

Висівкоподібний лишай Lichen bran

Лейкодерма Leucoderma

150 / 200
14-річний хлопець скаржиться на зміни кольору сечі - 'м’ясних помиїв’.’ З анамнезу відомо, що 2 тижні тому переніс тонзило-фарингіт. Об’єктивно: пастозність обличчя, повік. Артеріальний тиск - 135/90 мм рт.ст. Хлопець активний, не втрачає свідомості. В аналізі сечі: питома вага - 1025, еритроцити - вкривають все поле зору, білок - 0,165 г/л, еритроцитарні циліндри. Поставте попередній діагноз: A 14-year-old boy complains of changes in the color of his urine - 'meat scum'.' It is known from the anamnesis that he had tonsillopharyngitis 2 weeks ago. Objectively: pastiness of the face, eyelids. Blood pressure - 135/90 mm Hg. The guy is active, does not lose consciousness. In the urine analysis: specific gravity - 1025, erythrocytes - cover the entire field of vision, protein - 0.165 g/l, erythrocyte cylinders. Make a preliminary diagnosis:

Гострий пієлонефрит Acute pyelonephritis

Гострий постстрептококовий гломерулонефрит Acute poststreptococcal glomerulonephritis

Сечокам’яна хвороба Urolithiasis

Нефротичний синдром Nephrotic Syndrome

Гострий тубулоінтерстиціальний нефрит Acute tubulointerstitial nephritis

151 / 200
Породілля 22-х років на 4 добу післяпологового періоду скаржиться на головний біль, слабкість, біль в правій молочній залозі, різке підвищення температури тіла до 39°С, лихоманку. В ділянці верхньо-зовнішнього квадранта правої молочної залози пальпується щільний болісний інфільтрат, шкіра над яким гіперемована. Артеріальний тиск - 120/70 мм рт.ст., пульс - 110/хв. Живіт м’який, безболісний. Який імовірний діагноз? A 22-year-old woman in labor on the 4th day of the postpartum period complains of a headache, weakness, pain in the right mammary gland, a sharp increase in body temperature to 39°C, fever . In the area of the upper outer quadrant of the right mammary gland, a dense painful infiltrate is palpated, the skin over which is hyperemic. Blood pressure - 120/70 mm Hg, pulse - 110/min. The abdomen is soft, painless. What is the probable diagnosis?

Лактостаз Lactic Stasis

Гангренозний мастит Gangrenous Mastitis

Ретромаммарний абсцес Retromammary abscess

Лактаційний мастит, інфільтративна форма Lactation mastitis, infiltrative form

Флегмонозний мастит Pulmonary Mastitis

152 / 200
Жінка 26-ти років скаржиться на безпліддя протягом 3-х років. Чоловік здоровий. При обстеженні встановлено: тіло матки щільної консистенції, нормальних розмірів. Додатки не пальпуються. Проведена ме-тросальпінгографія - контрастна рідина у черевній порожнині з обох боків. Порожнина матки деформована. Припущено, що причина жіночого безпліддя - аномалія розвитку матки. Яке обстеження необхідно провести, щоб підтвердити діагноз? A 26-year-old woman has been complaining of infertility for 3 years. The man is healthy. During the examination, the body of the uterus is of a dense consistency, of normal size. The appendages are not palpable. Metrosalpingography was performed - contrast liquid in the abdominal cavity on both sides. The uterine cavity is deformed. It is assumed that the cause of female infertility is an abnormality in the development of the uterus. What examination should be performed to confirm the diagnosis?

Вишкрiбання стінок порожнини матки Scraping the walls of the uterine cavity

Зондування матки Probing of the uterus

Ультразвукове дослідження (УЗД) Ultrasound

Гістероскопія Hysteroscopy

Розширена кольпоскопія Advanced colposcopy

153 / 200
Серед прикріпленого населення лікаря загальної практики 30% складають люди похилого, старечого віку. Які особливості захворюваності цього контингенту повинен враховувати лікар для організації надання їм медичної допомоги? Among the attached population of a general practitioner, 30% are elderly people. What features of the morbidity of this contingent should the doctor take into account in order to provide them with medical care?

Велика кількість хронічних захворювань A large number of chronic diseases

Сезонний характер захворюваності та звернень за допомогою Seasonal nature of morbidity and requests for help

Велика кількість хвороб з нетяжким типовим перебігом A large number of diseases with a mild typical course

Значна частка неускладнених атипових захворювань A significant share of uncomplicated atypical diseases

Збільшення частки гострих інфекційних захворювань Increase in the share of acute infectious diseases

154 / 200
Мати дитини 1-го року скаржиться на постійний нав’язливий, частий, малопродуктивний кашель, іноді до блювання. Під час об’єктивного дослідження у пацієнта спостерігається прискорене дихання, помірне втя-гнення нижніх міжреберних м’язів, збільшення передньозаднього розміру грудної клітки. Під час респіраторних інфекцій з’являється бронхообструкція. У періоді новонародже-ності переніс меконіальний ілеус. Оберіть першочергове обстеження: The mother of a 1-year-old child complains of constant intrusive, frequent, low-productive cough, sometimes leading to vomiting. During an objective examination, the patient has rapid breathing , moderate involvement of the lower intercostal muscles, an increase in the anteroposterior size of the chest. During respiratory infections, broncho-obstruction appears. During the newborn period, he suffered meconial ileus. Choose the primary examination:

Рентгенографія органів грудної клітки X-ray of chest organs

Дослідження на хламідії та мікоплазму Research on chlamydia and mycoplasma

Генетичне тестування Genetic testing

Дослідження хлоридів поту Sweat Chloride Study

Комп’ютерна томографія легень Computed tomography of the lungs

155 / 200
У хворої 45-ти років з мегалобластною анемією при ендоскопічному дослідженні з біопсією слизової оболонки шлунка виявлені атрофічні ділянки дифузного характеру в фундальній частині шлунка. Який провідний механізм розвитку даного захворювання? In a 45-year-old patient with megaloblastic anemia, endoscopic examination with a biopsy of the gastric mucosa revealed atrophic areas of a diffuse nature in the fundal part of the stomach. What is the leading mechanism of the development of this disease? ?

Застосування НПЗП Use of NSAIDs

Променеве ураження Radiation damage

Аутоімунний (антитіла до обкладочних клітин) Autoimmune (antibodies to epithelial cells)

Хімічне ураження Chemical damage

Н. pylori - інфекція N. pylori - infection

156 / 200
Чоловік 63-х років був госпіталізований 5 днів тому з приводу інфаркту міокарда. Раптом він поскаржився на сильний біль у грудях та втратив свідомість. Пульс не визначається, тони серця відсутні. ЕКГ: синусовий ритм, QS та підйом сегмента ST в V 1-У4. Реанімаційні заходи неефективні. При пункції виявлена кров в порожнині перикарда. Вкажіть найбільш імовірне ускладнення: A 63-year-old man was hospitalized 5 days ago due to a myocardial infarction. He suddenly complained of severe chest pain and lost consciousness. The pulse is not detectable, heart sounds absent. ECG: sinus rhythm, QS and elevation of the ST segment in V 1-U4. Resuscitation measures are ineffective. Blood was detected in the pericardial cavity during puncture. Specify the most likely complication:

Порушення цілісності стінки лівого шлуночка Violation of the integrity of the wall of the left ventricle

Гострий перикардит Acute pericarditis

Тромбоемболія легеневої артерії Thromboembolism of the pulmonary artery

Гостра мітральна недостатність Acute mitral regurgitation

Порушення цілісності міжшлуночкової перегородки Disruption of the integrity of the interventricular septum

157 / 200
Хвора 55-ти років скаржиться на випорожнення 3-4 рази протягом 1-2 годин переважно вранці, після сніданку. Посилення діареї пов’язує з підвищеними вимогами з боку оточення, хвилюванням, очікуванням небезпеки. Об’єктивно: метушлива, багатомовна, збільшеної ваги. Шкіра без змін, тургор нормальний, живіт м’який безболісний. В аналізі крові змін не має; аналіз калу: несформова-ний, незначна кількість крохмальних зерен та м’язових волокон, лейкоцити - 3-4 в п/з; колоноскопія без патології. З метою лікування слід призначити: A 55-year-old patient complains of defecation 3-4 times during 1-2 hours, mainly in the morning, after breakfast. The increase in diarrhea is associated with increased demands from surroundings, excitement, anticipation of danger. Objectively: fussy, multi-lingual, increased weight. Skin unchanged, turgor normal, abdomen soft and painless. Blood analysis shows no changes; stool analysis: unformed, insignificant amount of starch grains and muscle fibers, leukocytes - 3-4 in p/z; colonoscopy without pathology. For the purpose of treatment, the following should be prescribed:

Дротаверин Drotaverin

Анаприлін Anaprilin

Лоперамід Loperamide

Ампіцилін Ampicillin

Бісакодил Bisacodyl

158 / 200
Хвора 25-ти років скаржиться на наявність декількох щільних, болючих вузлів у правій пахвовій ділянці, підвищення температури тіла до 38°С, загальну слабкість. Хворіє 3 доби. Локально: в правій пахвовій ділянці є три вузлоподібних утворення, які підвищуються над поверхнею шкіри, розмірами від 0,5 см до 1,5 см в діаметрі, різко болючі при пальпації, з чіткими контурами, шкіра над ними багрово-синюшного відтінку, підняття верхньої кінцівки обмежене через біль. Поставте діагноз: A 25-year-old patient complains of the presence of several dense, painful nodes in the right armpit, an increase in body temperature to 38°C, general weakness. She has been ill for 3 days. Locally: in the right axillary area there are three nodular formations that rise above the surface of the skin, the size of which is from 0.5 cm to 1.5 cm in diameter, sharply painful on palpation, with clear contours, the skin above them has a purplish-bluish tint, raising the upper limbs are limited due to pain. Make a diagnosis:

Гнійний лімфаденіт Suppurative lymphadenitis

Фурункул Furnish

Абсцес Abscess

Карбункул Carbuncle

Гнійний гідраденіт Suppurative hidradenitis

159 / 200
У пацієнтки 48-ми років, яка скаржиться на рясні та тривалі менструації, лікар на підставі даних ультразвукового дослідження (УЗД) припускає гіперплазію ендометрія. Які методи дослідження найбільш раціонально застосувати для уточнення діагнозу? In a 48-year-old female patient who complains of heavy and prolonged menstruation, the doctor suspects endometrial hyperplasia based on ultrasound examination data. What research methods are the most rational apply to clarify the diagnosis?

Кульдоскопія Culdoscopy

Кольпоскопія Colposcopy

Кольпоцитологія Colpocytology

Гістеросальпінгографія Hysterosalpingography

Гістероскопія, фракційне вишкрібання матки з наступним патогістологічним дослідженням Hysteroscopy, fractional scraping of the uterus followed by histopathological examination

160 / 200
При проведенні медичного огляду дітей дитячого будинку №5 було виявлено знижену працездатність, швидку втомлюваність, апатію, сонливість. У10% обстежених дітей спостерігається кровоточивість ясен, підшкірні точкові крововиливи. Нестача якого вітаміну виявляється в обстежених дітей? During the medical examination of the children of orphanage No. 5, reduced work capacity, quick fatigue, apathy, drowsiness were found. 10% of the examined children had bleeding gums, subcutaneous dot hemorrhages. Which vitamin is lacking in the examined children?

Вітамін E Vitamin E

Вітамін H Vitamin H

Вітамін B1 Vitamin B1

Вітамін D Vitamin D

Вітамін С Vitamin C

161 / 200
У дитини 11-ти років спостерігаються періодичні закрепи від народження, здуття живота, зниження апетиту, інколи нудота, гіпотрофія м’язів. На іригограмі спостерігається звуження в ділянці прямої кишки із супра-стенотичним розширенням ободової кишки вище цього місця. При аноманометрії аноре-ктальний рефлекс негативний. Який метод діагностики буде найбільш інформативним для підтвердження діагнозу? An 11-year-old child has periodic constipation since birth, abdominal distension, decreased appetite, sometimes nausea, muscle hypotrophy. On the irrigogram, there is a narrowing in the area of the rectus intestines with supra-stenotic expansion of the colon above this place. During anomanometry, the anorectal reflex is negative. What diagnostic method will be the most informative to confirm the diagnosis?

Ультразвукове дослідження (УЗД) Ultrasound

Біохімічне дослідження крові Biochemical blood test

Повношарова біопсія стінки прямої та ободової кишки Full-layer biopsy of the wall of the rectum and colon

Оглядова рентгенографія черевної порожнини Overview x-ray of the abdominal cavity

Термографія передньої черевної стінки Thermography of the anterior abdominal wall

162 / 200
До невропатолога звернувся хворий 54х років зі скаргами на тремор рук, головний біль, металевий присмак у роті, безсоння, підсилення слиновиділення. З анамнезу відомо, що він більше 10-ти років працює на заводі люмінесцентних ламп. Яким буде ваш діагноз? A 54-year-old patient turned to a neurologist with complaints of hand tremors, headache, metallic taste in the mouth, insomnia, increased salivation. From the anamnesis, it is known that he has more than 10 'You have been working at a fluorescent lamp factory for years. What will be your diagnosis?

Хронічна інтоксикація ртуттю Chronic mercury intoxication

Інтоксикація бензолом Benzene intoxication

Інтоксикація марганцем Manganese intoxication

Хронічна інтоксикація нітрофарбами Chronic intoxication with nitropaints

Хронічна інтоксикація свинцем Chronic lead intoxication

163 / 200
У хворого 52-х років, який звернувся до гастроентеролога з приводу виразки шлунка, виявлено диспластичне ожиріння, стриї на животі та стегнах. Артеріальний тиск -170/100 мм рт.ст. В ході лабораторного обстеження виявлено підвищений рівень кортизолу, помірно знижений рівень АКТГ Встановіть діагноз: A 52-year-old patient who consulted a gastroenterologist for a stomach ulcer was found to have dysplastic obesity, striae on the abdomen and thighs. Blood pressure -170/100 mm during the laboratory examination, an increased level of cortisol, a moderately decreased level of ACTH was found. Establish a diagnosis:

Гіпоталамічний синдром з ендокринно-обмінними порушеннями Hypothalamic syndrome with endocrine-metabolic disorders

Аліментарне ожиріння Alimentary obesity

Синдром Іценка-Кушинга Itsenko-Cushing syndrome

Хвороба Іценка-Кушинга Itsenko-Cushing disease

Синдром Золінгера-Еллісона Zollinger-Ellison Syndrome

164 / 200
Породілля 23-х років, пологи термінові нормальні. Через 2 доби після пологів встановлено: загальний стан задовільний. Пульс - 80/хв., артеріальний тиск - 120/70 мм рт.ст. Матка щільна, розташована на 3 пальця нижче пупка, безболісна. Виділення зі статевих шляхів помірні, кров’янисті. Який діагноз найбільш імовірний? 23-year-old woman in labor, term delivery normal. 2 days after delivery, the general condition was found to be satisfactory. Pulse - 80/min., blood pressure - 120/ 70 mm Hg. The uterus is dense, located 3 fingers below the navel, painless. Genital discharge is moderate, bloody. What is the most likely diagnosis?

Гематометра Hematometer

Післяпологовий метроендометрит Postpartum metroendometritis

Субінволюція матки Subinvolution of uterus

Фізіологічний післяпологовий період Physiological postpartum period

Лохіометра Lochiometer

165 / 200
Хвора 23-х років доставлена ургентно зі скаргами на біль внизу живота, інтенсив- ніший справа, з іррадіацією в пряму кишку, запаморочення. Вищезазначені скарги з’явилися раптово вночі. Остання менструація 2 тижні тому. Об’єктивно: шкірні покриви бліді, пульс - 92/хв., температура тіла - 36, 6°С, артеріальний тиск -100/60 мм рт.ст. Живіт дещо напружений, незначно болючий в нижніх відділах, симптоми подразнення очеревини слабо позитивні. Гемоглобін - 98 г/л. Яким буде попередній діагноз? A 23-year-old patient was brought urgently with complaints of pain in the lower abdomen, more intense on the right side, with radiation to the rectum, dizziness. The above-mentioned complaints appeared suddenly at night. Last menstruation 2 weeks ago. Objectively: skin is pale, pulse - 92/min., body temperature - 36.6°C, blood pressure -100/60 mm Hg. Abdomen is somewhat tense, slightly painful in lower parts, symptoms of peritoneal irritation are weakly positive. Hemoglobin - 98 g/l. What will be the preliminary diagnosis?

Ниркова колька Renal colic

Порушена позаматкова вагітність Disrupted ectopic pregnancy

Кишкова непрохідність Intestinal obstruction

Апоплексія яєчника Ovarian apoplexy

Гострий апендицит Acute appendicitis

166 / 200
У дитини, народженої при терміні геста-ції 39 тижнів, після надання реанімаційної допомоги з’явилися дихальні розлади, тяжкість яких зростала в динаміці. Навколоплідні води були забруднені меконієм. Частота дихання -90/хв., звертають на себе увагу значні ретра-кції, стогін на видиху, рухи крил носа, парадоксальне дихання. Вкажіть найбільш імовірну причину дихальних розладів у цієї дитини: A child born at 39 weeks of gestation developed respiratory disorders, the severity of which increased dynamically after resuscitation. The amniotic fluid was contaminated with meconium . Respiratory rate -90/min., attention is drawn to significant retractions, moaning on exhalation, movements of the wings of the nose, paradoxical breathing. Specify the most likely cause of respiratory disorders in this child:

Транзиторне тахіпное новонародженого Transient tachypnea of a newborn

Природжена вада серця Congenital heart defect

Респіраторний дистрес-синдром Respiratory distress syndrome

Синдром аспірації меконію Meconium aspiration syndrome

Природжена пневмонія Congenital pneumonia

167 / 200
До лікаря звернулася породілля на третю добу післяпологового періоду зі скаргами на біль в правій молочній залозі, утруднене зціджування молока, підвищення температури до 37,2°С. При огляді встановлено: локальне, помірно болісне, тверде утворення в ділянці правої молочної залози, шкіра над ним не змінена. Який діагноз буде найбільш імовірним? A woman in labor turned to the doctor on the third day of the postpartum period with complaints of pain in the right mammary gland, difficulty expressing milk, an increase in temperature to 37.2°C. During the examination established: a local, moderately painful, firm formation in the area of the right mammary gland, the skin above it has not changed. What diagnosis will be the most likely?

Серозний мастит Serous mastitis

Гнійний мастит Purulent mastitis

Гангренозний мастит Gangrenous Mastitis

Флегмонозний мастит Phlegmonous mastitis

Лактостаз Лактостаз

168 / 200
У хворого 52-х років ішемічна хвороба серця (ІХС): нестабільна (що вперше виникла) стенокардія. Стенозувальний коронаросклероз, СН І ст. Гіпертонічна хвороба ІІ ст., ст. 3, ризик 4. Зроблено стентування правої коронарної артерії. Кардіолог рекомендував терапію, що включає бета-адреноблокатор, подвійну антиагрегантну комбінацію (пла-вікс і кардіомагніл), гіполіпідемічні засоби (статини). Зазначте лабораторні показники, які Ви контролюватимете з метою безпечного тривалого застосування статинів: A 52-year-old patient has coronary heart disease (CHD): unstable (first-onset) angina pectoris. Stenosing coronary sclerosis, HF first stage. Hypertensive disease of the second stage. , stage 3, risk 4. Stenting of the right coronary artery was performed. The cardiologist recommended therapy that includes a beta-adrenergic blocker, a double antiplatelet combination (plavix and cardiomagnil), hypolipidemic agents (statins). Specify the laboratory parameters that you will monitor for the purpose safe long-term use of statins:

Рівень глюкози в крові Blood glucose level

Тригліцериди, бета-ліпопротеїди в сироватці крові Triglycerides, beta-lipoproteins in blood serum

Альфа-амілаза сироватки крові Serum alpha-amylase

Трансамінази Transaminases

Добова протеїнурія Daily proteinuria

169 / 200
Під час проведення реанімаційних заходів пацієнту 77-ми років, внаслідок ригідності грудної клітки не вдається виконувати компресії грудної клітки на глибину 5-6 см. Якими будуть Вашi дії? During resuscitation measures, a 77-year-old patient cannot perform chest compressions to a depth of 5-6 cm due to chest stiffness. What will be your actions?

Збільшити силу натискань не зважаючи на ризик перелому ребер Increase the force of pressing without taking into account the risk of fractured ribs

Зменшити силу натискань, щоб не допустити переломів ребер і груднини Reduce pressure to prevent rib and sternum fractures

Змістити точку натискань в сторону мечоподібного відростка Move the pressure point towards the xiphoid process

Збільшити частоту натискань до 150 разів на хвилину Increase the frequency of clicks to 150 times per minute

Продовжувати натискання з попередньою силою Continue pressing with previous force

170 / 200
Хворий 36-ти років внаслідок дорожньо-транспортної пригоди одержав важку черепномозкову травму та перебуває в коматозному стані. З діагностичною метою проведена ЕХО-енцефалографія, знайдений зсув серединних структур мозку вправо на 12 мм. За допомогою комп’ютерної томографії в лобово-тім’яній ділянці ліворуч виявлена субдуральна гематома розміром 6х7х7 см. Назвіть найбільш правильну тактику лікування: A 36-year-old patient received a severe craniocerebral injury as a result of a traffic accident and is in a comatose state. For diagnostic purposes, an echo-encephalography was performed, a shift of the middle structures of the brain was found to the right by 12 mm. With the help of computer tomography, a subdural hematoma measuring 6x7x7 cm was detected in the fronto-parietal area on the left. Name the most correct treatment tactics:

Застосування оксибаротерапії Use of oxybarotherapy

Проведення дезінтоксикаційної терапії Carrying out detoxification therapy

Проведення краніоцеребральної гіпотермії Craniocerebral hypothermia

Спостереження за хворим у динаміці Monitoring of the patient in dynamics

Хірургічне лікування Surgical treatment

171 / 200
Під час дослідження трупу громадянина 50-ти років виявлено: високо на шиї, вище рівня щитоподібного хряща, розташована странгуляційна борозна темно-коричневого кольору, яка йде знизу догори та справа наліво. Борозна відсутня на задній поверхні шиї, найкраще виражена на передній її поверхні, де глибина становить 0,2 см, а ширина -1,5 см. Трупні плями інтенсивні, фіолетового кольору, розташовані на нижніх кінцівках. Який вид механічної асфіксії мав місце? During the examination of the corpse of a 50-year-old citizen, it was found: high on the neck, above the level of the thyroid cartilage, there is a strangulation furrow of dark brown color, which goes from bottom to top and from right to left. The furrow is absent on the back of the neck, it is best expressed on its front surface, where the depth is 0.2 cm and the width is 1.5 cm. The cadaver spots are intense, purple in color, located on the lower extremities. What type of mechanical asphyxiation did he have place?

Закриття отворів рота та носа Closing mouth and nose

Задушення руками Hand Choked

Стиснення грудної клітки та живота Compression of chest and abdomen

Повішення Hanging

Задушення петлею Loop strangulation

172 / 200
Призначення глюкокортикостероїдних гормонів з метою передопераційної підготовки при дифузно-токсичному зобі необхідне для: The appointment of glucocorticosteroid hormones for the purpose of preoperative preparation for diffuse toxic goiter is necessary for:

Попередження анафілактичного шоку Anaphylactic shock warning

Попередження гострої недостатності на-днирників Warning of acute adrenal insufficiency

Попередження гіпопаратиреозу Prevention of hypoparathyroidism

Попередження тиреотоксичного кризу Thyrotoxic crisis warning

Попередження гіпотиреозу Hypothyroidism Warning

173 / 200
В організм людини з атмосферного повітря надходять декілька хімічних речовин. Як називається тип комбінованої дії, при якому сумісний її ефект перевищує суму ефектів кожної речовини, що входить у комбінацію, при їх ізольованій дії на організм? Several chemical substances enter the human body from atmospheric air. What is the type of combined action called, in which its combined effect exceeds the sum of the effects of each substance included in the combination, with their isolated effect on the body?

Антагонізм Antagonism

Комплексна дія Complex Action

Ізольована дія Isolated Action

Поєднана дія Combined Action

Потенціювання Potentiation

174 / 200
У хворого 67-ми років на 6-й день після планової операції раптово виникла гостра дихальна недостатність. Частота серцевих скорочень - 130/хв. Верхня половина тулуба, шия та обличчя ціанотичні, кашель, різкий біль за грудниною. Артеріальний тиск -90/60 мм рт.ст., центральний венозний тиск -330 мм вод.ст. На ЕКГ інверсія зубця Т Яке ускладнення розвинулося у хворого? A 67-year-old patient suddenly developed acute respiratory failure on the 6th day after a planned operation. Heart rate - 130/min. Upper half of the body, neck and the face are cyanotic, cough, sharp pain behind the sternum. Arterial pressure -90/60 mm Hg, central venous pressure -330 mm Hg. T-wave inversion on the ECG. What complication developed in the patient?

Тромбоемболія легеневої артерії Thromboembolism of the pulmonary artery

Гостра недостатність коронарних артерій Acute coronary artery insufficiency

Гіповолемічний шок Hypovolemic shock

Повторна кровотеча Recurrent bleeding

Гострий інфаркт міокарда Acute myocardial infarction

175 / 200
У хворого 20-ти років на наступний день після закритої травми голови раптово підсилився головний біль, виникли нудота, блювання, відчуття затерпання та слабкості лівої руки. В неврологічному статусі виявлений симптом Керніга, розширення правої зіниці та центральний парез лівої руки. Встановіть попередній клінічний діагноз: A 20-year-old patient suddenly developed a headache, nausea, vomiting, numbness and weakness of the left arm the day after a closed head injury. In the neurological status Kernig's sign, dilation of the right pupil and central paresis of the left hand are found. Make a preliminary clinical diagnosis:

Струс головного мозку Concussion

Забій головного мозку Brain stroke

Вторинний менінгіт Secondary meningitis

Абсцес головного мозку Brain abscess

Субдуральна гематома Subdural Hematoma

176 / 200
Хвора 35-ти років звернулася до лікаря зі скаргами на наявність новоутвору на шкірі правої гомілки. Об’єктивно: невус неправильної форми діаметром 1 см, без чітких меж, з сухою поверхнею, темно-коричневого кольору. Встановлено діагноз: імовірна меланома шкіри правої гомілки. Який метод верифікації діагнозу меланоми шкіри? A 35-year-old patient turned to the doctor with complaints about the presence of a neoplasm on the skin of the right lower leg. Objectively: an irregularly shaped nevus with a diameter of 1 cm, without clear boundaries, with a dry surface, dark brown in color. The diagnosis was established: probable melanoma of the skin of the right leg. What is the method of verification of the diagnosis of melanoma of the skin?

Інцизійна біопсія Incisional biopsy

Ексцизійна біопсія Excisional biopsy

Мазок-відбиток Smear-imprint

Позитронно-емісійна томографія - комп’ютерна томографія (ПЕТ-КТ) Positron emission tomography - computed tomography (PET-CT)

Ультразвукове дослідження (УЗД) Ultrasound

177 / 200
Хворий 38-ми років, водій автобуса, у робочий час внаслідок аварії отримав тяжку травму спинного мозку (є акт про нещасний випадок на виробництві). В результаті цього його визнано інвалідом першої групи. Визначте причину інвалідності: A 38-year-old patient, a bus driver, suffered a severe spinal cord injury during working hours as a result of an accident (there is an act of an accident at work). As a result, his recognized as a disabled person of the first group. Determine the reason for the disability:

Трудове каліцтво Occupational disability

Дорожньо-транспортна пригода Traffic accident

Професійне захворювання Occupational disease

Тяжкість травми Injury Severity

Загальне захворювання General disease

178 / 200
Людині циркулярною пилкою завдано поранення першого пальця правої кисті. Медична допомога надана в травмпункті. Дані амбулаторного хворого: ”Стан задовільний, артеріальний тиск - 90/60 мм рт.ст., пульс - 100/хв. Перший палець правої кисті відокремлений від основної фаланги і утримується за рахунок шкірного клаптя’’ На прийомі судово-медичного експерта: 'Перший палець правої кисті відсутній на рівні основної фаланги. Культю сформовано'. Визначте ступінь тяжкості тілесного ушкодження: A person was injured with a circular saw on the first finger of the right hand. Medical assistance was provided in the trauma center. Data of the outpatient: 'The condition is satisfactory, blood pressure - 90/60 mm Hg. Art., pulse - 100/min. The first finger of the right hand is separated from the main phalanx and is held by a skin flap.' At the appointment of the forensic medical expert: 'The first finger of the right hand is missing at the level of the main phalanx. The callus is formed.' Determine the degree of severity. bodily injury:

Дане тілесне ушкодження належить до середнього ступеня тяжкості за критерієм процентної втрати працездатності до 30% This injury belongs to the medium degree of severity according to the criterion of percentage loss of working capacity up to 30%

Дане тілесне ушкодження належить до середнього ступеня тяжкості за критерієм тривалості розладу здоров’я This physical injury belongs to the medium degree of severity according to the criterion of the duration of the health disorder

Дане тілесне ушкодження належить до тяжких тілесних ушкоджень за критерієм небезпеки для життя This injury belongs to severe injuries according to the criterion of danger to life

Дане тілесне ушкодження належить до категорії легких тілесних ушкоджень, що спричинили короткочасний розлад здоров’я This injury belongs to the category of light injuries that caused a short-term health disorder

Дане тілесне ушкодження належить до тяжких тілесних ушкоджень за критерієм процентної втрати працездатності більше 30% This injury belongs to severe injuries according to the criterion of percentage loss of working capacity of more than 30%

179 / 200
У хворого 56-ти років в ході ультразвукового обстеження встановлена наявність вегетацій на штучному мітральному клапані, який було імплантовано 3 роки тому, та підклапанних структурах з ознаками деструкції вегетацій та розвитку недостатності штучного клапану. Яке лікування показане хворому? In a 56-year-old patient, during an ultrasound examination, the presence of vegetations on the artificial mitral valve, which was implanted 3 years ago, and subvalvular structures with signs of vegetation destruction and development of artificial valve insufficiency. What treatment is indicated for the patient?

Обмеження фізичного навантаження Physical load limitation

Імплантація штучного водія ритму Implantation of an artificial pacemaker

Призначення вазодилататорів Prescription of vasodilators

Оперативне (репротезування) Operative (reprosthetics)

Подальше спостереження Follow-up

180 / 200
Хворий 57-ми років перебуває на лікуванні з приводу політравми. Через 8 діб виникли скарги на виражений кашель з виділенням гнійного харкотиння, виражену задишку, підвищення температури до 38,8°C. В анамнезі: хронічний бронхіт, палить 20 років. Об’єктивно: в легенях вислуховується жорстке дихання, в нижніх відділах правої легені - вологі дрібнопухирчасті хрипи. Проба на ВІЛ - негативна. Який діагноз у даного хворого? A 57-year-old patient is being treated for polytrauma. After 8 days, there were complaints of severe cough with purulent sputum, severe shortness of breath, temperature rise to 38, 8°C. History: chronic bronchitis, smoker for 20 years. Objectively: harsh breathing is heard in the lungs, moist fine-vesicular rales in the lower parts of the right lung. The HIV test is negative. What is the diagnosis of this patient?

Позагоспітальна пневмонія Community-acquired pneumonia

Нозокоміальна пневмонія Nosocomial pneumonia

Пневмонія осіб з імунодефіцитом Pneumonia of persons with immunodeficiency

Інфекційне загострення бронхіту Infectious exacerbation of bronchitis

Аспіраційна пневмонія Aspiration pneumonia

181 / 200
Вночі у хворого з’явилися: головний біль, біль у правому оці з іррадіацією в праву скроню, нудота, блювання, зниження зору правого ока, туман, райдужні кола навколо джерела світла. Об’єктивно: праве око - застійна ін’єкція судин склери, рогівка набрякла, її чутливість знижена, передня камера неглибока, зіниця широка. Рефлекс з очного дна тьмяно-рожевий, деталі не офтальмоско-пуються через набряк рогівки. Внутрішньоо-чний тиск - 58 мм рт.ст. Поставте діагноз: At night, the patient developed: headache, pain in the right eye with radiation to the right temple, nausea, vomiting, decreased vision in the right eye, fog, iridescent circles around the light source. Objectively: the right eye - congestive injection of scleral vessels, the cornea is swollen, its sensitivity is reduced, the anterior chamber is shallow, the pupil is wide. The reflex from the fundus is dull pink, the details are not ophthalmoscopied due to corneal edema. Internally blood pressure - 58 mm Hg Make a diagnosis:

Іридоцикліт Iridocyclite

Гострий напад глаукоми Acute attack of glaucoma

Бактеріальний кон’юнктивіт Bacterial conjunctivitis

Набухаюча катаракта Swelling Cataract

Відкритокутова глаукома Open-angle glaucoma

182 / 200
На прийом до педіатра звернулась мати із 4-и місячною дитиною зі скаргами на підвищення температури до 37,5°C. Дитина народжена доношеною, 8-9 балів за шкалою Апгар. При аналізі медичної документації виявлено, що у дитини немає щеплення проти туберкульозу. В які часові терміни повинно було проведене щеплення проти туберкульозу? A mother with a 4-month-old child came to the pediatrician with complaints of an increase in temperature to 37.5°C. The child was born full-term, 8-9 points for by the Apgar scale. During the analysis of the medical documentation, it was found that the child has not been vaccinated against tuberculosis. When should the vaccination against tuberculosis have been carried out?

Протягом перебування в пологовому будинку після попередньої проби Манту During the stay in the maternity hospital after the previous Mantoux test

Протягом перших 24 годин життя During the first 24 hours of life

Протягом першого тижня життя During the first week of life

Доношена дитина, не пізніше 48 годин від народження Full-term baby, no later than 48 hours after birth

Доношена дитина, 3-5 доба життя Full-term baby, 3-5 days old

183 / 200
Головний лікар районної лікарні видав наказ про порядок проведення обов’язкових планових профілактичних щеплень. Які контингенти населення підлягають обов’язковим плановим щепленням? The chief physician of the district hospital issued an order on the procedure for conducting mandatory scheduled preventive vaccinations. Which population groups are subject to mandatory scheduled vaccinations?

Особи з окремих професійних груп ризику (єгері, мисливці тощо) Persons from certain occupational risk groups (hunters, hunters, etc.)

Особи, які могли бути інфіковані Persons who may have been infected

Діти, яким оформлюють документи у дитячий садочок або школу Children for whom documents are drawn up for kindergarten or school

Особи, які мали контакт з джерелом інфекції в епідемічному осередку Persons who had contact with the source of infection in the epidemic center

Діти, підлітки та дорослі, які досягли відповідного віку Children, teenagers and adults who have reached the appropriate age

184 / 200
Жінка 65-ти років хвора на хронічне обструктивне захворювання легень із загостреннями хвороби щонайменше двічі на рік. Останнє загострення з підвищенням температури та гнійним харкотинням було проліковане 3 тижні тому. Яка тактика ведення хворої щодо вторинної імунопрофілактики загострень та прогресування дихальної недостатності? A 65-year-old woman suffers from chronic obstructive pulmonary disease with exacerbations of the disease at least twice a year. The last exacerbation with fever and purulent sputum was treated 3 weeks ago. What are the patient management tactics for secondary immunoprophylaxis of exacerbations and progression of respiratory failure?

Вакцинація БЦЖ після негативного результату проби Манту BCG vaccination after a negative Mantoux test

Призначення гомологічного імуноглобулі-ну Prescription of homologous immunoglobulin

Вакцинація проти гемофільної інфекції типу В Vaccination against hemophilic infection type B

Усі щеплення протипоказані All vaccinations are contraindicated

Вакцинація проти пневмококу та проти грипу Pneumococcal and influenza vaccination

185 / 200
Першовагітна 25-ти років звернулася у січні до відділення патології вагітних у терміні 27 тижнів із ознаками тяжкої гострої респіраторної інфекції. Встановлено та лабораторно підтверджено діагноз 'грип' Стан вагітної поступово погіршувався, хвора переведена до відділення інтенсивної терапії, розпочата штучна вентиляція легень (ШВЛ). Незважаючи на лікування, відбулася антенатальна загибель плода. Який підхід із перелічених нижче, є найбільш дієвим у попередженні цього ускладнення вагітності? In January, a 25-year-old pregnant woman turned to the department of pathology of pregnant women at 27 weeks with signs of a severe acute respiratory infection. The diagnosis of 'flu' was established and laboratory confirmed. The condition of the pregnant woman gradually worsened, the patient was transferred to the intensive care unit, and artificial lung ventilation (VLT) was started. Despite treatment, antenatal fetal death occurred. Which of the following approaches is most effective in preventing this pregnancy complication?

Прийом противірусних препаратів під час епідемії грипу Taking antiviral drugs during an influenza epidemic

Застосування імуномодуляторів Use of immunomodulators

Відмовитись від відвідування буд-яких скупчень людей під час епідемії Refuse to visit any gatherings of people during an epidemic

Застосування деескалаційної антибіотико-терапії відразу після появи перших проявів гострої респіраторної вірусної інфекції (ГРВІ) Application of de-escalation antibiotic therapy immediately after the appearance of the first manifestations of acute respiratory viral infection (ARVI)

Вакцинація від грипу усіх вагітних та жінок, що планують вагітність Influenza vaccination of all pregnant women and women planning to become pregnant

186 / 200
Дитина 4-х років здорова та невакцино-вана проти кору. В регіоні, де вона мешкає зафіксований спалах кору. Лікар призначив першу дозу вакцини КПК (проти кору, паротиту та краснухи). Коли даній дитині потрібно ввести другу дозу вакцини? A 4-year-old child is healthy and unvaccinated against measles. In the region where she lives, an outbreak of measles has been recorded. The doctor prescribed the first dose of the PDA vaccine (against measles, mumps and rubella) When does this child need a second dose of the vaccine?

Через 6 місяців After 6 months

Через 3 місяці After 3 months

В 14 років життя At 14 years old

Через 1 місяць In 1 month

В 6 років життя 6 years old

187 / 200
До сімейного лікаря звернулася учениця 14-ти років зі скаргами на подряпини на обличчі, які наніс сусідський кіт 2 дні тому. При огляді подряпини поверхневі, вкриті струпом. Виконано первинну хірургічну обробку ран. Відомо що дитина отримала всі планові щеплення згідно з календарем. Якими будуть подальші дії? A 14-year-old student came to the family doctor with complaints about scratches on her face, which were caused by a neighbor's cat 2 days ago. On examination, the scratches are superficial, covered with a scab. Done initial surgical treatment of wounds. It is known that the child received all scheduled vaccinations according to the calendar. What will be the next steps?

Введення антирабічного імуноглобуліну Introduction of anti-rabies immunoglobulin

Введення антирабічної вакцини + антира-бічного імуноглобуліну Introduction of anti-rabies vaccine + anti-rabies immunoglobulin

Введення антирабічної вакцини + правцевий анатоксин Introduction of rabies vaccine + tetanus toxoid

Спостереження за твариною Animal Observation

Введення антирабічної вакцини Introduction of anti-rabies vaccine

188 / 200
Вагітну жінку 33-х років у терміні 1617 тижнів вагітності вкусив собака у ділянку шиї. Пацієнтка доставлена бригадою швидкої допомоги в травмпункт. Загальний стан середньої тяжкості. Артеріальний тиск -100/60 мм рт.ст., пульс - 92/хв. При огляді рана в ділянці шиї з рваними краями й помірною кровотечею. Якою буде тактика лікаря? A 33-year-old pregnant woman at 1617 weeks of pregnancy was bitten by a dog in the neck area. The patient was taken by an ambulance to the trauma center. The general condition is moderate. Blood pressure - 100/60 mm Hg, pulse - 92/min. When examining a wound in the neck area with ragged edges and moderate bleeding. What will the doctor's tactics be?

Переривання вагітності за медичними показами після обробки рани Termination of pregnancy according to medical indications after wound treatment

Зупинка кровотечі, рішення про вакцинацію при ознаках сказу у тварини Stopping of bleeding, decision on vaccination for signs of rabies in an animal

Хірургічна обробка рани, введення протиправцевого анатоксину та вакцини від сказу Surgical treatment of the wound, administration of tetanus toxoid and rabies vaccine

Хірургічна обробка рани, спостереження за собакою, рішення про профілактику сказу через 10 діб Surgical treatment of the wound, observation of the dog, decision on rabies prevention in 10 days

Зупинка кровотечі, хірургічна обробка рани Stop bleeding, surgical wound treatment

189 / 200
Дівчинка народилася від I вагітності, I вагінальних пологів у терміні гестації 39 тижнів (маса 3350 г, довжина тіла 53 см). 17-річна мати дитини ВЩ-позитивна. Стан дитини після народження задовільний. Коли можна провести вакцинацію БЦЖ даній дитині? A girl was born from the 1st pregnancy, the 1st vaginal delivery at a gestation period of 39 weeks (weight 3350 g, body length 53 cm). The 17-year-old mother of the child is BSC-positive . The child's condition after birth is satisfactory. When can BCG vaccination be given to this child?

На 3-тю добу On the 3rd day

Після проби Манту After Mantoux test

На 1-шу добу On the 1st day

Після уточнення ВЩ-статусу дитини After clarifying the child's health status

Не щеплювати в жодному випадку Do not vaccinate in any case

190 / 200
Новонароджена недоношена дитина народилася з масою 1900 г. Переведена у відділення патології новонароджених для подальшого лікування з діагнозом 'перинатальна гіпоксична енцефалопатія” Щеплення БЦЖ в пологовому будинку не проводилося. Коли слід щеплювати дану дитину проти туберкульозу? A newborn premature baby was born with a weight of 1900 g. She was transferred to the Department of Neonatal Pathology for further treatment with a diagnosis of 'perinatal hypoxic encephalopathy' BCG vaccination was not carried out in the maternity hospital. When should this child be vaccinated against tuberculosis?

По досягненні маси тіла 2500 г Upon reaching a body weight of 2500 g

Після усунення ознак неврологічної патології After eliminating signs of neurological pathology

Не щеплювати дитину взагалі Do not vaccinate the child at all

В 1 рік життя In 1 year of life

У віці 6 місяців Aged 6 months

191 / 200
У хірургічний кабінет звернувся хлопець 17-ти років, зі скаргами на наявність рубаної рани правої стопи. Із анамнезу відомо, що травму отримав сокирою під час роботи по господарству. Всі щеплення отримував відповідно віку. Об’єктивно: температура тіла - 36,8oC, пульс - 68/хв., артеріальний тиск -120/75 мм рт.ст. Які дії лікаря? A 17-year-old boy came to the surgery with complaints of a cut wound on his right foot. From the anamnesis, it is known that he was injured with an ax while working on the farm. He received all vaccinations according to his age. Objectively: body temperature - 36.8oC, pulse - 68/min., blood pressure -120/75 mm Hg. What actions did the doctor take?

Ввести протиправцеву сироватку Enter anti-tetanus serum

Ввести правцевий анатоксин Inject Right Toxoid

Ввести правцевий анатоксин та протиправцевий імуноглобулін Enter tetanus toxoid and anti-tetanus immunoglobulin

Динамічне спостереження Dynamic monitoring

Ввести правцевий анатоксин з дифтерійним анатоксином Enter tetanus toxoid with diphtheria toxoid

192 / 200
Пацієнтка 30-ти років впродовж 5-ти років хворіє на системний червоний вовчак (СЧВ) і щоденно приймає 16 мг метилпре-днізолону. Яке твердження щодо вакцинації проти грипу у даному випадку є вірним? A 30-year-old patient has been suffering from systemic lupus erythematosus (SLE) for 5 years and has been taking 16 mg of methylprednisolone daily. What is the statement about influenza vaccination is y correct in this case ?

Вакцинація після зменшення дози преднізолону до мінімальної Vaccination after reducing the dose of prednisolone to the minimum

Вакцинація протипоказана хворим, що отримують глюкокортикоїди Vaccination is contraindicated in patients receiving glucocorticoids

Рекомендована щорічна планова вакцинація Recommended annual routine vaccination

Вакцинація проводиться під прикриттям антиалергічних препаратів Vaccination is carried out under the guise of anti-allergic drugs

Будь-яка вакцинація протипоказана Any vaccination is contraindicated

193 / 200
Хворий 67-ми років з хронічною лімфо-лейкемією отримує курс хіміотерапії. Якими будуть рекомендації щодо вакцинації проти грипу за умови достатнього рівня гранулоцитів та лімфоцитів у периферійній крові? A 67-year-old patient with chronic lymphocytic leukemia is receiving a course of chemotherapy. What would be the recommendations for influenza vaccination if there are sufficient levels of granulocytes and lymphocytes in the peripheral blood?

Не раніше, ніж через 1 тиждень після хіміотерапії Not earlier than 1 week after chemotherapy

Через 1 рік після хіміотерапії 1 year after chemotherapy

Не раніше, ніж через 4 тижні після хіміотерапії Not earlier than 4 weeks after chemotherapy

Взагалі протипоказана Generally contraindicated

Можна під час хіміотерапії Possible during chemotherapy

194 / 200
Жінка 23-х років, вагітна 12 тижнів, у якої в анамнезі оперативне втручання з при- воду тетради Фалло у віці 1-го року, щорічно отримувала вакцинацію проти грипу, остання -1 рік тому. Які рекомендації повинен дати сімейний лікар щодо вакцинації проти грипу цього року? A 23-year-old woman, 12 weeks pregnant, with a history of surgical intervention due to tetralogy of Fallot at the age of 1, received annual vaccination against flu, the last - 1 year ago. What recommendations should the family doctor give regarding vaccination against the flu this year?

Вакцинація після пологів Vaccination after childbirth

Відкласти вакцинацію до третього триместру Postpone vaccination until the third trimester

Цього року вакцинацію не проводити No vaccination this year

Провести вакцинацію інактивованою вакциною Vaccinate with an inactivated vaccine

Провести вакцинацію рекомбінантною вакциною Vaccinate with recombinant vaccine

195 / 200
У лікаря-стоматолога 26-ти років при обстеженні виявили помірну гіперферменте-мію, позитивний тест на HBsAg. Його дружина 24-х років не має щеплення від вірусного гепатиту В. Дитина 2-х років належно вакцинована за віковим графіком. Яка профілактика вірусного гепатиту В повинна бути проведена у родині? A 26-year-old dentist was diagnosed with moderate hyperenzymemia during the examination, a positive test for HBsAg. His 24-year-old wife is not vaccinated against viral hepatitis B. A 2-year-old child is properly vaccinated according to the age schedule. What prevention of viral hepatitis B should be carried out in the family?

Призначення дружині специфічної хіміо-профілактики ненуклеозидними аналогами Prescribing specific chemoprophylaxis to the wife with non-nucleoside analogs

Призначення дружині та дитині специфічної хіміопрофілактики ненуклеозидними аналогами Prescribing specific chemoprophylaxis to the wife and child with non-nucleoside analogues

Негайна вакцинація дружини при негативному тесті на HBsAg Immediate vaccination of wife with a negative HBsAg test

Призначення специфічного імуноглобуліну дружині та дитині Prescribing specific immunoglobulin to wife and child

Призначення специфічного імуноглобуліну дитині Prescribing specific immunoglobulin to a child

196 / 200
Чоловік 56-ти років скаржиться на швидку втомлюваність, погіршення апетиту протягом останнього місяця, що пов’язує з перевантаженням на роботі. Постійної сім’ї не має, повідомляє про сумнівні статеві зв’язки. Замолоду протягом декількох років був донором. Обстежений сімейним лікарем. Виявлено РНК до вірусу гепатиту С методом ПЛР, ступінь фіброзу згідно зі шкалою METAVIR за даними фіброскану - F3. Які щеплення лікар повинен запропонувати хворому? A 56-year-old man complains of rapid fatigue, loss of appetite during the last month, which is associated with overload at work. He has no permanent family, reports about dubious sexual relations. He was a donor for several years when he was young. He was examined by a family doctor. RNA for the hepatitis C virus was detected by the PCR method, the degree of fibrosis according to the METAVIR scale according to the fibroscan data is F3. What vaccinations should the doctor offer the patient?

Всі щеплення протипоказані All vaccinations are contraindicated

Щеплення проти вірусних гепатитів А та В Vaccination against viral hepatitis A and B

Щеплення проти грипу щорічно Flu vaccination annually

Щеплення проти пневмококу, вірусного гепатиту В та щорічно проти грипу Vaccination against pneumococci, viral hepatitis B and annually against influenza

Щеплення проти пневмококу, вірусних гепатитів А, В та щорічно проти грипу Vaccination against pneumococci, viral hepatitis A, B and annually against influenza

197 / 200
Дитина 6-ти місяців народжена ВІЛ-інфікованою матір’ю. ВІЛ-статус дитини не визначений. При огляді: стан задовільний. Фізичний та нервово-психічний розвиток відповідає віку. Проведена вакцинація в 2 місяці та 4 місяці проти дифтерії, правця, кашлюка, поліомієліту. Згідно з календарем профілактичних щеплень в Україні лікар призначив продовження вакцинації проти даних інфекцій. Яка вакцина ПРОТИПОКАЗАНА даній дитині? A 6-month-old child was born to an HIV-infected mother. The child's HIV status is not determined. On examination: the condition is satisfactory. Physical and neuropsychological development corresponds to age. Vaccination was carried out at 2 months and 4 months against diphtheria, tetanus, whooping cough, poliomyelitis. According to the calendar of preventive vaccinations in Ukraine, the doctor prescribed continuation of vaccination against these infections. Which vaccine is CONTRAINDICATED for this child?

Оральна (жива) поліомієлітна вакцина Oral (live) polio vaccine

АКДП-вакцина DPT vaccine

Інактивована вакцина проти поліомієліту Inactivated polio vaccine

- -

Вакцинація не проводиться до визначення ВІЛ-статусу Vaccination is not performed until HIV status is determined

198 / 200
Дівчина 15-ти років захворіла на кір. Відомо, що вона страждає на бронхіальну астму з високим рівнем загального IgE та на залізодефіцитну анемію: рівень НЬ- 110 г/л, вміст сироваткового заліза 6,6 мкмоль/л (нормальний показник - 10,6-33,6 мкмоль/л). Щеплена вакциною проти кору, краснухи, паротиту одноразово в віці 1 року. Який фактор зумовив захворювання дитини? A 15-year-old girl fell ill with measles. It is known that she suffers from bronchial asthma with a high level of total IgE and iron-deficiency anemia: Hb level - 110 g/ l, the content of serum iron is 6.6 μmol/l (normal indicator - 10.6-33.6 μmol/l). She was vaccinated against measles, rubella, mumps once at the age of 1 year. What factor caused the child's illness?

Сидеропенія Sideropenia

Анемія Anemia

Пубертатний вік Puberty age

Неповна вакцинація Incomplete vaccination

Атопія Атопія

199 / 200
Чоловік 26-ти років, який ”живе з ВІЛ/СНЩ” з приводу чого отримує анти-ретровірусну терапію, звернувся до лікаря з приводу планового щеплення. Пацієнт вакцинований згідно з календарем щеплень. Лікар призначив щеплення проти дифтерії та правця. До якого класу належать дані вакцини? A 26-year-old man who 'lives with HIV/TMD' and receives antiretroviral therapy, consulted a doctor about a routine vaccination. The patient is vaccinated according to the vaccination calendar. The doctor prescribed vaccination against diphtheria and tetanus. What class do these vaccines belong to?

Анатоксини Anatoxins

Корпускулярні живі Corpuscular live

Корпускулярні вбиті (інактивовані) Corpusculars killed (inactivated)

Хімічні Chemical

Рекомбінантні Recombinant

200 / 200
Жінка 58-ми років з нейросенсорною глухотою внаслідок ускладнення після перенесеного гнійного менінгіту планує встановлення кохлеарного імплантату. Щеплення проти якої інфекції необхідно зробити для попередження інфекційних захворювань центральної нервової системи і органів слуху? A 58-year-old woman with sensorineural deafness due to a complication after purulent meningitis plans to install a cochlear implant. Vaccination against which infection should be done to prevent infectious diseases of the central nervous system and organs of hearing?

Пневмококової інфекції Pneumococcal infection

Поліомієліту Polio

Туберкульозу після негативного результату проби Манту Tuberculosis after a negative Mantoux test

Вірусу кліщового енцефаліту Tick-borne encephalitis virus

Гемофільної інфекції Haemophilus infection